172
©™ ©™ Colen Publishing www.colenpublishing.com

Comprehensive Neurosurgery Board Review Flash cards

Embed Size (px)

DESCRIPTION

High yield comprehensive neuroscience flash cards with board style question and referenced explained answers. This review is all inclusive of all aspects of the neurosurgery boards; includes neurology, pathology, anatomy, critical care, neurosurgery, radiology and biology.Ideal for use when studying advanced neuroscience as a student or resident. Faculty would benefit using these flash cards as a quick refresher of high-yield topics in neurology.Carry 10-15 cards in your pocket and study from these cards to utilize your time spent while waiting for an elevator, lunch line, or on the ward.Please visit our website: www.colenpublishing.com for more information.

Citation preview

Page 1: Comprehensive Neurosurgery Board Review Flash cards

©™©™

Colen Publishing

www.colenpublishing.com

Page 2: Comprehensive Neurosurgery Board Review Flash cards

©™©™

Colen Publishing, L.L.C.PO Box 36536Grosse Pointe Woods, MI 48236Author and Editor: Chaim B. Colen, M.D., Ph.D.Editorial Assistant: Roxanne E. Colen, PA-C

COPYRIGHT © 2008 by Colen Publishing, L.L.C. This book, including all parts thereof, is legally protected by copyright. Any use, exploitation, or commercialization outside the narrow limits set by copyright legislation without the author’s consent if illegal and liable to prosecution. This applies in particular to photostat reproduction, copying, mimeographing or duplication of any kind, translating, preparation of microfilms, and electronic data processing and storage.Some of the product names, patents, and registered designs referred to in this book are in fact registered trademarks or proprietary names even though specific reference to this fact is not always made in the text. Therefore, the appearance of a name without designation as proprietary is not to be construed as a representation by the publisher that it is in the public domain. Permissions may be sought directly from Colen Publishing, L.L.C. by writing to the above address.Printed in ChinaColen Flash-Review: Neurosurgery, 2nd EditionISBNVolume 1: 1-935345-01-X Volume 2: 1-935345-02-8 2 Volume Set: 1-935345-00-1

Note: Knowledge in medicine is constantly changing. The author has consulted sources believed to be reliable in the effort to provide information that is complete and in accord with the standards at the time of publication. However, in view of the possibility of human error by the author in preparation of this work, warrants that the information contained herein is in every respect accurate and complete, and that the author is not responsible for any errors or omissions or for the results obtained from use of such information. The reader is advised to confirm the information contained herein with other sources. This is especially important in connection with new or infrequently used drugs. In such instances, the product information sheet included in the package with each drug should be reviewed.

Colen Publishing

Page 3: Comprehensive Neurosurgery Board Review Flash cards

©™©™

Glossary

COPYRIGHT-------------------------------------------------- 1PREFACE------------------------------------------------------ 1HOW TO USE THIS CARD REVIEW-------------------- 1CONTRIBUTORS-------------------------------------------- 4GLOSSARY--------------------------------------------------- 1NEUROSURGERY------------------------------------------ 110NEUROLOGY ------------------------------------------------ 86NEUROPATHOLOGY-------------------------------------- 238NEUROANATOMY----------------------------------------- 57NEUROCRITICAL CARE---------------------------------- 80NEURORADIOLOGY--------------------------------------- 73NEUROBIOLOGY------------------------------------------- 64 BONUS BIOSTATISTICS---------------------------------- 6

Page 4: Comprehensive Neurosurgery Board Review Flash cards

©™©™

Preface• The idea to undertake such a large Flashcard review spawned from watching my wife Roxanne

study for her Physician Assistant Boards. Diligently every day she would create a set of 7-10 flashcards from her study material that she would take with her to work. Later on, when I was studying for my written Neurosurgery Board examination, I gleaned information from various texts and other study guides and wrote down the most relevant material on cards for quick review while at work. It was amazing how much time during the day would be available to review these cards. If there was a delay in a OR case, a long lunch-line, a traffic jam (especially the i94 on a Friday afternoon) or waiting for my wife at her OB/GYN appointment -these little cards were specially handy. Always ambitious in life, the thought of giving this study tool to the busy neurosurgery resident was captivating. My expectation is to enable the resident with a quick yet informative review of basic neuroscience principles. With positive encouragement from my fellow residents on the 1st edition, I cautiously proceed here with updating information, adding new images, improved illustrations and clarification of neuroscience concepts. May this endeavor serve to better our wonderful science inherited through the legacy of Harvey Cushing, Neurosurgery.

Chaim September 9, 2008

Page 5: Comprehensive Neurosurgery Board Review Flash cards

©™©™

The Colen Flash-Review

Author and EditorChaim B. Colen, M.D., Ph.D.Department of Neurological SurgeryWayne State University School of MedicineDetroit, Michigan

Assistant EditorRoxanne E. Colen, M.S., PA-CColen Publishing, LLCGrosse Pointe, Michigan

Page 6: Comprehensive Neurosurgery Board Review Flash cards

©™©™

AcknowledgementsI would like to give thanks to a great many wonderful persons whose efforts, although not inscribed in

these cards, were instrumental in making this monumental task possible. One exceptional individual to whom I owe special thanks is my mother in-in-law, Colleen Johns, who babysat my daughter Emily and son Joshua for hours on end, while my wife and I toiled through hundreds of pages of various textbooks and journal articles, formatted questions, and drew computer illustrations. To my daughter Emily Rivka, who incessantly tugged at my pants trying to get my attention to the squirrel in our backyard ;and that big bright smile from my son Joshua that continually sent me optimism. To Mahmoud and Abhi who spent hours at my home assisting with typing, researching and editing; Naomi whose positive attitude in life is exceptionally brightening and uplifted the group’s 2 am brainstorming sessions when I still had to wake up early to work the next day, all the pathologists, especially Doha, who assisted in taking photographs, Dr. William Kupsky, for allowing us access to his collection of unique neuropathology, and to all the medical students especially Kristyn, whose hard work is admirable. There are those whose names are not here but did assist in some way, thank you. I am forever indebted to my training program, the Wayne State University neurosurgery program, my Chairman Dr. Murali Guthikonda, and Associate Chairman Dr. Setti S. Rengachary whose moral support over the last five years has kept me on this educational drive. For this second edition, there were fellow residents that gave me input and new insight that has helped to improve this edition over the first.

To my parents Joseph and Leila, educators of true dedicated quality, and to whom I owe my homeschooling education and self-motivation. Lastly to my wife Roxanne, whose patience with my ambitiousness knows no boundaries.

Thank you All,Chaim September 9, 2008

Page 7: Comprehensive Neurosurgery Board Review Flash cards

©™©™

How to use this Flashcard review

• These cards are intended to cover most of the aspects of the Neurosurgery Board Examination. They are not a COMPLETE review and therefore they are not intended to replace textbooks. We would advise using these cards during the last couple of weeks before your board exam except for the pathology section which you should go through all year to better remember the photographs in it (heavily encountered during the boards!). BOARD FAVORITEquestions are of extreme importance and most likely to bump into during the boards, so make you sure you know how to answer them right.

• Good luck!• Chaim B. Colen, M.D., Ph.D.

Page 8: Comprehensive Neurosurgery Board Review Flash cards

©™©™

Faculty Reviewers

Murali Guthikonda, MD Professor and Chairman Department of Neurological SurgeryWayne State University School of MedicineDetroit, Michigan

Setti Rengachary, MD Associate Chairman Department of Neurological SurgeryWayne State University School of MedicineDetroit, Michigan

William, J. Kupsky, MDDepartment of Neuropathology Wayne State University School of MedicineDetroit, Michigan

Page 9: Comprehensive Neurosurgery Board Review Flash cards

©™©™

• With ever increasing scope and complexity of knowledge base, the current day trainee or practitioner of neurosurgery finds it difficult to keep up with the explosion of neurosurgical information. This is compounded by a healthy growth in specialization in various branches of neurosurgery.

• Chaim has made an attempt to make life simpler by incorporating small quanta of knowledge on flashcards accompanied by clear and simple illustrations. The user may review as few or as many cards as his/her time will allow. Although not meant to be substitutes for standard comprehensive texts and atlases, these cards help to refresh the information learned from the bedside, operating room and standard books. Each card represents a mini-examination with instant access to appropriate answers.

• This is a fun way to recall neurosurgical information especially before an upcoming test.

Setti S. Rengachary, M.D.Department of Neurological Surgery

Forward

Page 10: Comprehensive Neurosurgery Board Review Flash cards

©™©™

Physician Contributing Authors

Mahmoud Rayes, MDDepartment of Neurological Surgery WSU School of Medicine

Erika Peterson, MDUT Southwestern,Department of Neurological Surgery Dallas, Texas

Rivka R. Colen, MDDepartment of RadiologyThe Massachusetts General HospitalHarvard Medical SchoolBoston, Massachusetts

Doha Itani, MDDepartment of PathologyWSU School of MedicineDetroit, Michigan

Page 11: Comprehensive Neurosurgery Board Review Flash cards

©™©™

Contributing Medical Students

Darmafall, KristynWayne State UniversitySchool of MedicineClass of 2012

Davis, Naomi Wayne State UniversitySchool of Medicine Class of 2011

Dub, LarissaWayne State UniversitySchool of MedicineClass of 2012

Faulkiner, RodneyWayne State UniversitySchool of MedicineClass of 2012

Galinato, AnthonyWayne State UniversitySchool of MedicineClass of 2012

Gotlib, DorothyWayne State UniversitySchool of MedicineClass of 2009

Kozma, BonitaWayne State UniversitySchool of MedicineClass of 2008

Lai, Christopher Wayne State UniversitySchool of MedicineClass of 2010

Larson, SarahWayne State UniversitySchool of MedicineClass of 2012

Martinez, DerekWayne State UniversitySchool of MedicineClass of 2011

Matthew SmithWayne State UniversitySchool of MedicineClass of 2011

Matto, ShereenWayne State UniversitySchool of MedicineClass of 2012

Page 12: Comprehensive Neurosurgery Board Review Flash cards

©™©™

Contributing Undergraduates

Jeffrey P. KallasWayne State UniversityClass of 2010

Abhinav KrishnanWayne State UniversityClass of 2010

Peter PaximadisWayne State UniversityClass of 2008

Page 13: Comprehensive Neurosurgery Board Review Flash cards

©™©™

• This cerebellar specimen is consistent with:

A. Lewy bodyB. Neuritic plaqueC. Kuru plaqueD. Hirano body

NeuropathologyQ?

2

Page 14: Comprehensive Neurosurgery Board Review Flash cards

©™©™

Neuropathology SectionNeuropathologyA.• The correct answer is C, Kuru plaque.• Pathology of variant Creutzfeldt-Jakob

disease – Kuru demonstrating a “kuru plaque” in cerebellum.

• Accumulation of prion protein PrP in the form of amyloid plaques occurs in some forms of prion disease.

• Hirano bodies (rod-shaped eosin inclusions) are seen in Alzheimer’s disease.

Cerebellum: granular cell layer

PAS

Kuru Plaque

Liberski PP. Amyloid plaques in transmissible spongiform encephalopathies (prion diseases). Folia Neuropathol. 2004;42 Suppl B:109-19. Review.

Cerebellar Infection, Prion, Creutzfeldt-Jakob disease, Kuru, amyloid

Page 15: Comprehensive Neurosurgery Board Review Flash cards

©™©™

• This brain specimen is most consistent with:

A. Lewy bodyB. Neuritic plaqueC. Kuru plaqueD. Hirano body

NeuropathologyQ?

3

Page 16: Comprehensive Neurosurgery Board Review Flash cards

©™©™

NeuropathologyA.• The correct answer is D, Hirano body.• Hirano bodies (rod-shaped eosin

inclusions of mostly actin filaments) are seen in Alzheimer’s disease.

• Accumulation of prion protein PrP in the form of amyloid plaques occurs in the pathology of variant Creutzfeldt-Jakob disease – Kuru demonstrates “kuru plaques” in the cerebellum.

• Lewy bodies are seen in Parkinson’s disease and diffuse Lewy body disease. These are formed from α-synuclein.

Kuljis RO: Modular corticocerebral pathology in Alzheimer's disease. In: Mangone CA, Allegri RF, Ariza, eds. Dementia: A Multidisciplinary Approach. 1997: 143-55.

Section of hippocampus

Hirano body

BOARD FAVORITE!

Degenerative disease, Alzheimer’s disease, Hirano bodies

Page 17: Comprehensive Neurosurgery Board Review Flash cards

©™©™

Used with permission from Handbook of Neurosurgery andNeurology in Pediatrics; By Chaim B. Colen, MD, PhD.

• What is the most likely diagnosis seen in this fundoscopic examination?

A. Drusen bodiesB. Retinopathy of diabetesC. Retinal hemorrhageD. Pseudopapilledema

NeuropathologyQ?

9

Page 18: Comprehensive Neurosurgery Board Review Flash cards

©™©™

NeuropathologyA.• The correct answer is C, retinal hemorrhages.• Shaken baby syndrome was described by Caffey as a clinical triad: subdural

hematomas, subarachnoid hemorrhages, and retinal hemorrhages.• "Shaken baby syndrome" (SBS) results in intracranial and intraocular hemorrhages

with no evidence of external trauma. The cause of these injuries is vigorous shaking of an infant being held by the chest, shoulders, or extremities.

• Physicians are mandated to report cases of child abuse, elder abuse, sexual abuse, domestic violence, and assault. Physicians are also required to notify the authorities if anyone has a specific plan to commit suicide or homicide.

• Drusen bodies aka “pseudopapilledema” are colloid or hyaline bodies lying beneath the retinal pigment epithelium. They may occur either secondary to changes in the choroid that affect the pigment epithelium or as an autosomal dominant disorder of the retinal pigment epithelium.

Martin HA, Woodson A, Christian CW, Helfaer MA, Raghupathi R, Huh JW. Shaken baby syndrome. Crit Care Nurs Clin North Am. 2006 Sep;18(3):279-86. Review.

Shaken baby syndrome, Pediatrics, Retinal hemorrhages, subdural hematomas,

subarachnoid hemorrhages

Page 19: Comprehensive Neurosurgery Board Review Flash cards

©™©™

• Which pairing is most accurate for the localization of medulloblastomas in the pediatric and adult population?

A. Children : Midline, Adults : MidlineB. Children : Lateral, Adults : LateralC. Children : Lateral, Adults : MidlineD. Children : Midline, Adults : Lateral

NeuropathologyQ?

54

Page 20: Comprehensive Neurosurgery Board Review Flash cards

©™©™

Homer-Wright rosette –(pseudorosette) radially arranged nuclei surrounding central fibrillary material (red arrow); histopathological features of medulloblastoma.

NeuropathologyA.• The correct answer is D, children :

midline, adults: lateral

MRI brain -contrast enhanced sagittal image showing the midline location of the tumor in a pediatric patient.

BOARD FAVORITE!

Neoplasm, Cerebellum, PNET, medulloblastoma, location of medulloblastoma, Homer-Wright

rosette

Page 21: Comprehensive Neurosurgery Board Review Flash cards

©™©™

• Compared to pilocytic astrocytoma:

A. This tumor has a better prognosis

B. This tumor has a similar prognosis.

C. This tumor has a worse prognosis

D. Prognosis is irrelevant

NeuropathologyQ?

114

Page 22: Comprehensive Neurosurgery Board Review Flash cards

©™©™

NeuropathologyA.• The correct answer is C, this tumor has a worse prognosis..• Gemistocytic astrocytoma is WHO grade II and is distinctive in its appearance, having

astrocytes with generous eosinophilic cytoplasm with a “stuffed” stellate appearance. • Low-grade astrocytomas with a significant fraction of gemistocytes progress more rapidly

and typically carry a p53 mutation.

Krouwer HG, Davis RL, Silver P, Prados M. Gemistocytic astrocytomas: a reappraisal. J Neurosurg. 1991 Mar;74(3):399-406.

Secondary GBMProgenitor cell

pRB loss, NK4α increase, PTEN, loss

Secondary GBM

PDGFR increase

P53 loss

Grade IIastro

Grade IIIastro

BOARD FAVORITE!

neoplasm, astrocytoma, gemistocytic astrocytoma, WHO grade III

Gemistocytic astrocytoma

Page 23: Comprehensive Neurosurgery Board Review Flash cards

©™©™

• This muscle specimen (Gomori stain) is consistent with which disorder?

A. Myoclonic seizuresB. Disuse atrophyC. Steroid atrophyD. Denervation atrophy

NeuropathologyQ?

138

Page 24: Comprehensive Neurosurgery Board Review Flash cards

©™©™

NeuropathologyA.• The correct answer is A, myoclonic

seizures.• Mitochondrial disorders (e.g.

MELAS, MERRF) clinically are associated with seizures and muscle biopsy shows ragged-red fibers and rimmed vacuoles.

• Type 1 fiber atrophy- think muscle disease! -myotonic dystrophy and congenital myopathy

• Type 2 muscle atrophy- think neurogenic or muscle disease! –denervation, disuse, myasthenia gravis, steroids and paraneoplastic syndrome (BOARD FAVORITE!)

Fukuhara N, Tokiguchi S, Shirakawa K, Tsubaki T. Myoclonus epilepsy associated with ragged-red fibres (mitochondrial abnormalities ): disease entity or a syndrome? Light-and electron-microscopic studies of two cases and review of literature. J Neurol Sci. 1980 Jul;47(1):117-33.

RAGGED RED FIBERS RIMMED VACUOLE

BOARD FAVORITE!

Muscle, Mitochondrial disease, seizures, ragged-red fibers, rimmed vacuoles, type 1 fiber atrophy,

type 2 fiber atrophy

Page 25: Comprehensive Neurosurgery Board Review Flash cards

©™©™

• This peripheral nerve biopsy is consistent with:

A. Myasthenia gravisB. Onion bulb formationC. Wallerian degenerationD. Neurofibroma

NeuropathologyQ?

139

Page 26: Comprehensive Neurosurgery Board Review Flash cards

©™©™

NeuropathologyA.• The correct answer is B, onion bulb

formation.• Onion bulb formation occurs

after repeated cycles of demyelination and remyelination result in a thick layer of abnormal myelin around the peripheral axons. These changes cause what is referred to as an onion bulb appearance.

• May be seen in a few disorders: Charcot-Marie-Tooth (CMT) and Refsum’s disease (phytanic acid disorder).

Bornemann A, Hansen FJ, Schmalbruch H: Nerve and muscle biopsy in a case of hereditary motor and sensory neuropathy type III with basal lamina onion bulbs. Neuropathol Appl Neurobiol 1996 Feb; 22(1): 77-81.

ONION BULBSONION BULB (electron microscopy)

BOARD FAVORITE!

Peripheral nerve, Onion bulb formation, myelin, Charcot-Marie-Tooth, Refsum’s disease

Page 27: Comprehensive Neurosurgery Board Review Flash cards

©™©™

• Symptoms of a patient with this diagnosis might be alleviated with which of the following drugs:

A. Retroviral drugB. Anticholinesterase inhibitorC. Beta-interferonD. Trimethoprim-

sulfamethoxazole

NeuropathologyQ?

140

Page 28: Comprehensive Neurosurgery Board Review Flash cards

©™©™

NeuropathologyA.• The correct answer is A,

anticholinesterase inhibitor.• Microscopy shows an amyloid

plaque and neurofibrillary tangles that would be consistent with the diagnosis of Alzheimer’s disease.

• Donepezil is a synthetic noncovalent reversible inhibitor of acetylcholinesterase (AChE) for the treatment of mild to moderate dementia associated with Alzheimer's disease.

• Other drugs include galantamine and rivastigmine.

Kuljis RO: Modular corticocerebral pathology in Alzheimer's disease. In: Mangone CA, Allegri RF, Ariza, eds. Dementia: A Multidisciplinary Approach. 1997: 143-55.

Neurofibrillary tangle

Amyloid plaque

Degenerative disease, Dementia, Alzheimer’s disease, neurofibrillary tangles, amyloid plaque,

Donepezil, Acetylcholinesterase inhibitor

Page 29: Comprehensive Neurosurgery Board Review Flash cards

©™©™

• This brain specimen is most consistent with:

A. Alzheimer’s diseaseB. Parkinson’s diseaseC. Pick’s diseaseD. Wilson’s disease

NeuropathologyQ?

142

Page 30: Comprehensive Neurosurgery Board Review Flash cards

©™©™

NeuropathologyA.• The correct answer is B,

Parkinson’s disease.• Microscopy shows melanophages

and neuronal cell loss in the substantia nigra pars compacta.

• Lewy bodies are also seen. These are formed from α-synuclein.

• Pick bodies are seen in Pick’s disease using silver stain.

Hodaie M, Neimat JS, Lozano AM. The dopaminergic nigrostriatal system and Parkinson's disease: molecular events in development, disease, and cell death, and new therapeutic strategies. Neurosurgery. 2007 Jan;60(1):17-28; discussion 28-30.

NEURONAL LOSS LEWY BODY

BOARD FAVORITE!

Degenerative disease, Movement disorder, Parkinson’s disease, melanophages, lewy bodies,

alpha-synuclein

Page 31: Comprehensive Neurosurgery Board Review Flash cards

©™©™

• This neonatal brain specimen is consistent with which hemorrhage grade?

A. Grade IB. Grade IIC. Grade IIID. Grade IV

NeuropathologyQ?

148

Page 32: Comprehensive Neurosurgery Board Review Flash cards

©™©™

NeuropathologyA.• The correct answer is C, grade III.• This gross specimen demonstrates germinal matrix

hemorrhage (GMH) and intraventricular hemorrhage (IVH) and hydrocephalus which are the most common and most important neurologic injuries in preterm neonates.

• Premature infants lack the ability to autoregulate the cerebral blood pressure. Fluctuations in cerebral blood pressure and flow can rupture the primitive germinal matrix vessels or lead to infarction of the metabolically active germinal matrix. Neurologic sequelae, include cerebral palsy, mental retardation, and seizures.

Levy ML, Masri LS, McComb JG: Outcome for preterm infants with germinal matrix hemorrhage and progressive hydrocephalus. Neurosurgery 1997 Nov; 41(5): 1111-7; discussion 1117-8.

GRADE 3

IVH and hydrocephalus

GMH

Grading SystemGrade 1 - confined GMHGrade 2 - IVH without hydrocephalus.Grade 3 - IVH with associated hydrocephalusGrade 4 - Intraparenchymal hemorrhage

BOARD FAVORITE!

Vasculature, hemorrhage, hemorrhage grading, germinal matrix hemorrhage, intraventricular

hemorrhage, Pediatrics

Page 33: Comprehensive Neurosurgery Board Review Flash cards

©™©™

• This pathology is most likely due to:

A. Vitamin A toxicityB. Cyanocobalamine

deficiency C. Thiamine deficiencyD. Vitamin B6 deficiencyE. Alcohol intake

NeuropathologyQ?

149

Page 34: Comprehensive Neurosurgery Board Review Flash cards

©™©™

NeuropathologyA.• The correct answer is C, thiamine

deficiency.• Thiamine deficiency is seen in chronic

malnourished alcoholics. Note the old infarction that has occurred in the mamillary bodies.

• Wernicke-Korsakoff’s syndrome –subacute amnesia seen in damage to the mamillary bodies and may be caused by thiamine deficiency and may be reversible with the administration of thiamine.

Torvik A. Topographic distribution and severity of brain lesions in Wernicke's encephalopathy. Clin Neuropathol. 1987 Jan-Feb;6.

Old infarction

BOARD FAVORITE!

Nutritional deficiency, Thiamine deficiency, Wernicke-Korsakoff syndrome, mamillary bodies

Page 35: Comprehensive Neurosurgery Board Review Flash cards

©™©™

• This brain specimen is most consistent with which disease?

A. Wilson’s (hepatolenticular degeneration)

B. Alzheimer’sC. Pick’sD. Parkinson’sE. Huntington’s

NeuropathologyQ?

151

Page 36: Comprehensive Neurosurgery Board Review Flash cards

©™©™

NeuropathologyA.• The correct answer is A, Wilson's disease

(hepatolenticular degeneration). • Wilson’s disease is caused by accumulation

of copper in the brain with decreased serum copper and ceruloplasmin and increased urine copper. Kayser-Fleischer rings form around the iris.

• Microscopy demonstrates Alzheimer type 2 astrocytes which have a large clear vesicular nuclei and prominent nucleoli. Proliferation of large protoplasmic astrocytes such as Opalski cells may be seen.

• Research suggests that these reactive astrocytes may be involved in the process of copper detoxification in WD.

Bertrand E, Lewandowska E, Szpak GM, Hoogenraad T, Blaauwgers HG, Czlonkowska A, Dymecki J. Neuropathological analysis of pathological forms of astroglia in Wilson's disease. Folia Neuropathol. 2001;39(2):73-9.

BOARD FAVORITE!

Degenerative disease, Wilson’s disease, Alzheimer type 2 astrocytes, Opalski cells,

Copper

Page 37: Comprehensive Neurosurgery Board Review Flash cards

©™©™

• This muscle biopsy specimen is consistent with:

A. Congenital myopathyB. Myotonic dystrophyC. Steroid myopathyD. Mitochondrial myopathy

Note: Type 1 fibers labeled (1).

NeuropathologyQ?

152

1

Page 38: Comprehensive Neurosurgery Board Review Flash cards

©™©™

NeuropathologyA.• The correct answer is C, steroid myopathy.• Steroid myopathy is characterized by

type 2 myofiber atrophy!

• Type 1 fiber atrophy- think muscle disease! -myotonic dystrophy and congenital myopathy

• Type 2 muscle atrophy- think neurogenic or muscle disease! –denervation, disuse, myasthenia gravis, steroids and paraneoplastic syndrome (BOARD FAVORITE)

STEROID MYOPATHY

TYPE 2 ATROPHY

Griggs RC, Mendell JR, Miller RG: Congenital myopathies. In: Evaluation and Treatment of Myopathies. Philadelphia: FA Davis Co; 1995: 211-46.

BOARD FAVORITE!

Muscle, myopathy, steroid myopathy, type 2 fiber atrophy

Page 39: Comprehensive Neurosurgery Board Review Flash cards

©™©™

• This gross brain specimen is most consistent with:

A. AdrenoleukodystrophyB. Multiple sclerosisC. Mitochondrial myopathyD. Canavan’s disease

NeuropathologyQ?

154

Page 40: Comprehensive Neurosurgery Board Review Flash cards

©™©™©™

NeuropathologyA.• The correct answer is B, multiple sclerosis.• Multiple sclerosis (MS) is an idiopathic

(autoimmune?) inflammatory demyelinating disease of the CNS. Animal model experimental allergic encephalomyelitisdemonstrated by autoimmunity to myelin basic proteins. (BOARD FAVORITE)

• Episodes of de/remyelination result in a chronic burned-out plaque with relative preservation of axons and gliosis (recently axon transection has been reported in acute exacerbations).

• Incidence is higher in Caucasians. Female-to-male ratio is 2:1.

• Classic presentation -optic neuritis, transverse myelitis, internuclear ophthalmoplegia, paresthesias

Noseworthy JH, Lucchinetti C, Rodriguez M, Weinshenker BG: Multiple sclerosis. N Engl J Med 2000 Sep 28; 343(13): 938-52.

Periventricular location

Chronic plaques

BOARD FAVORITE!

Demyelinating disease, Inflammatory disease, Multiple Sclerosis, myelin basic protein

Page 41: Comprehensive Neurosurgery Board Review Flash cards

©™©™

• The gross spinal cord specimen (A) is most consistent with:

A. AdrenoleukodystrophyB. Multiple sclerosisC. Amyotrophic lateral sclerosisD. Neurofibromatosis

NeuropathologyQ?

A B

156

Page 42: Comprehensive Neurosurgery Board Review Flash cards

©™©™

Neuropathology

A B

A.• The correct answer is C, amyotrophic

lateral sclerosis.• Amyotrophic lateral sclerosis

progressive loss of upper and lower motor neurons.

• Pathology: progressive loss of anterior horn cells leads to atrophy of ventral nerve fibers and of skeletal muscles; progressive loss of primary motor neurons in motor cortex leads to degeneration of corticospinal/ corticobulbar tracts.

• Adrenoleukodystrophy (ALD)-leukodystrophy is an x-linked recessive disease due to deficiency in peroxisomal lipid oxidation.

Gartner J, Braun A, Holzinger A , et al: Clinical and genetic aspects of X-linked adrenoleukodystrophy. Neuropediatrics 1998 Feb; 29(1): 3-13.

Ventral nerve root atrophy

BOARD FAVORITE!

Normal ventral roots

spinal cord, Degenerative disease, Amyotrophic lateral sclerosis, Upper motor neuron, Lower

motor neuron

Page 43: Comprehensive Neurosurgery Board Review Flash cards

©™©™

• This gross brain specimen shows atrophy that is consistent with:

A. Multiple sclerosisB. Pick’s diseaseC. Alzheimer’s diseaseD. Acute cerebral infarction

NeuropathologyQ?

157

Page 44: Comprehensive Neurosurgery Board Review Flash cards

©™©™

NeuropathologyA.• The correct answer is Alzheimer’s

disease.• Alzheimer’s disease is the most

common dementing illness in adults, characterized by progressive dementia over several years. There is increased frequency with increasing age and in familial cases, usually earlier onset.

• Note the diffuse brain atrophy of Alzheimer's disease, unlike Pick’s disease which has mostly frontal lobe atrophy.

• Risk is increased in Down’s syndrome (BOARD FAVORITE). Kuljis RO: Modular corticocerebral pathology in Alzheimer's disease. In: Mangone CA,

Allegri RF, Ariza, eds. Dementia: A Multidisciplinary Approach. 1997: 143-55.

Frontal Atrophy

Temporal Atrophy

BOARD FAVORITE!

Degenerative disease, Alzheimer’s disease, dementia, diffuse atrophy

Page 45: Comprehensive Neurosurgery Board Review Flash cards

©™©™

• The pathology seen here is most likely due to:A. Cerebellar astrocytomaB. AlcoholismC. TraumaD. Ruptured aneurysmE. Arteriovenous malformation

NeuropathologyQ?

158

Page 46: Comprehensive Neurosurgery Board Review Flash cards

©™©™

NeuropathologyA.• The correct answer is….• To obtain the answer to this question and to

view over 400 more comprehensive pathology questions please purchase the full product here!

Bolla L, Palmer RM: Paraneoplastic cerebellar degeneration. Case report and literature review. Arch Intern Med 1997 Jun 9; 157(11): 1258-62.

BOARD FAVORITE!

Degeneration, Cerebellum, Alcohol related degeneration, superior vermis of cerebellum

Page 47: Comprehensive Neurosurgery Board Review Flash cards

©™

• When the interpeduncular cistern is filled with blood secondary to a ruptured posterior communicating aneurysm, which of the following choices describes the safest surgical approach?A. Superior to the posterior communicating artery, because there is only one

perforating arterial branch and so this area is safeB. Opening the membrane of Lilliequist above the posterior communicating artery to

avoid injury to the perforators that may be covered with bloodC. Opening the membrane of Lilliequist below the posterior communicating artery to

avoid injury to the superiorly directed perforators that may be covered with bloodD. The membrane of Lilliequist should not be opened

NeurosurgeryQ?

2

Page 48: Comprehensive Neurosurgery Board Review Flash cards

©™

• The correct answer is C. It is safer to open the membrane of Lilliequist below the PComA to avoid injury to the superiorly directed perforators that may be covered with blood.

• The posterior communicating artery (PComA) usually gives off 2–10 perforators which begin approximately 2 to 3 mm from its origin. Branches, the anterior thalamoperforating arteries, run postero-medially into the interpeduncular cistern and supply the inferior optic chiasm, optic tract, tuber cinereum, mammillary bodies, subthalamus, posterior hypothalamus, and the anterior thalamus.

• An important point is that aneurysms almost always arise from the superior medial aspect of the PComA so that when the interpeduncular cistern is filled with blood, in case of a ruptured intracranial aneurysm, it is safer to open the membrane of Lilliequist below the PComA to avoid injury to these superiorly directed perforators that may be covered with blood. Of these perforators, the largest and most constant one is referred as the premamillary artery or thalamotuberal artery.

NeurosurgeryA.

Zeal AA, Rhoton AL Jr. Microsurgical anatomy of the posterior cerebral artery. J. Neurosurg. 1978 (48) 534–559.

Classification: Neurosurgery, Ruptured Posterior Communicating Aneurysm,

Surgical Approach

Page 49: Comprehensive Neurosurgery Board Review Flash cards

©™

• Which of the following choices best describes the Wartenberg’s sign? A. Ulnar nerve compressionB. Adduction of the index fingerC. Weakness of the third palmar interosseous muscleD. Inability to flex the thumb

NeurosurgeryQ?

8

Page 50: Comprehensive Neurosurgery Board Review Flash cards

©™

NeurosurgeryA.• The correct answer is C, weakness of the third palmar

interosseous muscle• Wartenberg’s sign is one of the earliest signs of ulnar

nerve entrapment. It is the result of the weakness of the third palmar interosseous muscle, which leads to abducted posture of the small finger.

A. M. Richards. Key Notes on Plastic Surgery. Blackwell Publishing. 2002. p. 215.H. Richard Winn, M.D. Youman’s Neurological Surgery 5th Edition. Philadelphia, PA: Elsevier 2004. p. 3925.

Wartenberg’s sign

• Ulnar nerve compression locations:• Upper arm: arcade of Struthers – aponeurotic band anterior to

the medial head of the triceps (NOT STRUTHER’S LIGAMENT which is a ligament connecting the medial epicondyle and the supracondylar process).

• Elbow/Forearm: in the cubital tunnel between the two heads of the flexor carpi ulnaris. MOST COMMON LOCATION (see neuroanatomy section)

• Wrist: Guyon’s canal (see neuroanatomy section)

BOARD FAVORITE!

Classification: Neurosurgery, Clinical findings, Wartenberg’s Sign

Page 51: Comprehensive Neurosurgery Board Review Flash cards

©™

• When should an open craniotomy be used to eliminate a vein of Galen malformation in an infant or older child?

A. Yasargil I and IIB. Yasargil II and IVC. Yasargil I and IVD. Yasargil III and V

NeurosurgeryQ?

23

Page 52: Comprehensive Neurosurgery Board Review Flash cards

©™

NeurosurgeryA.• The correct answer is A, open craniotomy should be used with Yasargil types I and II.• Vein of Galen aneurysmal malformation is a rare congenital anomaly of the cerebral circulation. It

makes up approximately 1% of intracranial vascular malformations, but they represent 30% of pediatric intracranial vascular malformations.

• Two methods of classifications had been used extensively in the literature. The first classification divided it into mural and choroidal type. The choroidal type is where there are multiple shunts communicating with the anterior dilatation of the median prosencephalic vein. The mural type is where the arteriovenous shunt is in the wall of the dilated vein. The second classification described by Yasargil divides into type I to IV in which type I corresponds to mural types and type II and III correspond to the choroidal type.

C.A. Raybaud, C.M. Strother, J.K. Hald. Aneurysms of the vein of Galen: embryonic considerations and anatomical features relating to the pathogenesis of the malformation. Neuroradiology 1989;31:109-128.H. Richard Winn, M.D. Youman’s Neurological Surgery 5th Edition. Philadelphia, PA: Elsevier 2004. p. 3442.

Angiogram showing a mural type vein of Galen aneurysmal malformation supplied by the right posterior choroidal artery.

vein of Galen aneurysmal malformation

Classification: Neurosurgery, Vein of Galen Malformation, Open Craniotomy

Page 53: Comprehensive Neurosurgery Board Review Flash cards

©™

• According to this cervical x-ray which of the following statements is most accurate?

A. You must surgically fuse this fracture.B. Most of these fractures will heal without

surgical fusionC. It is likely due to flexion injuryD. It is due to fracture of the pedicles

NeurosurgeryQ?

24

Page 54: Comprehensive Neurosurgery Board Review Flash cards

©™

NeurosurgeryA.• The correct answer is B. Most hangman’s fractures heal with

immobilization and do not warrant surgical intervention. In general, type 1 injuries are treated with a rigid collar for 8 to 12 weeks. Type 2 injuries are initially reduced with traction in slight extension followed by a halo vest for 12 weeks. A halo vest in compression and slight extension is used for type 2A injuries. Immediate surgery is required for type 3 injuries to stabilize the C2/C3 facet joint.

• The hangman's fracture consists of bilateral pars interarticularisfractures involving the C2 vertebral body. Anterior subluxation or dislocation of the C2 vertebral body will often occur in consequence to a hangman’s fracture, which itself occurs secondary to hyperextension injuries seen during automobile accidents or hangings.

Levine AM, Edwards CC. The management of traumatic spondylolisthesis of the axis. J Bone Joint Surg Am.1985;67:217-226.

BOARD FAVORITE!

Classification: Neurosurgery, Hangman’s Fracture, Treatment

Page 55: Comprehensive Neurosurgery Board Review Flash cards

©™

• In order to be directly above the foramen of Monro, where would the corpus callosum need to be incised?

A. At the genuB. 1 cm behind the genuC. 2.5 cm behind the genuD. 5 cm behind the genuE. 2.5 cm anterior to the splenium

NeurosurgeryQ?

45

Page 56: Comprehensive Neurosurgery Board Review Flash cards

©™

NeurosurgeryA.• The correct answer is C. The foramen of Monro is directly beneath the corpus

callosum at around 2.5 cm behind the genu.

2 cm

right hemisphere

genu of the corpus callosum

foramen of Monro

Classification: Neurosurgery, Corpus Callosum Incision, Foramen of Monro

Page 57: Comprehensive Neurosurgery Board Review Flash cards

©™

• The most appropriate next step in management for the abnormality seen in this image would be to:A. EmbolizeB. Inject alcoholC. Consider coilingD. Consider surgeryE. No intervention at this time

NeurosurgeryQ?

46

Page 58: Comprehensive Neurosurgery Board Review Flash cards

©™

NeurosurgeryA.• The correct answer is E, no intervention at this time.• The angiogram seen here depicts the classical findings seen in venous

malformations (VM), caput medusa. VM consist of a large cortical draining veinreceiving a collection of medullary veins that usually occurs near the angle of the ventricle. They rarely hemorrhage. Thus, when diagnosed, they do not require intervention.

• 30% are associated with cavernous malformations.• There may be multiple angiomas in blue nevus syndrome.

Perrini P, Lanzino G. The association of venous developmental anomalies and cavernous malformations: pathophysiological, diagnostic, and surgical considerations. Neurosurg Focus. 2006 Jul 15;21(1):e5. Review.

Classification: Neurosurgery, Venous Malformations, Management

Page 59: Comprehensive Neurosurgery Board Review Flash cards

©™

• The strength of a screw is proportional to:A. The length LB. The outside diameter (thread diameter)3

C. The inside diameter (ID)3

D. The product of the ID and length squared (ID x L) 2

E. All of the above

NeurosurgeryQ?

65

Page 60: Comprehensive Neurosurgery Board Review Flash cards

©™

NeurosurgeryA.• The correct answer is C, the inside

diameter (ID)3

ID

OD

Classification: Neurosurgery, Screw Strength

Page 61: Comprehensive Neurosurgery Board Review Flash cards

©™

• The order of structures traversed when doing Deep Brain Stimulation (DBS):A. Zona incerta, thalamus, subthalamic nucleus, substantia nigraB. Thalamus, zona incerta, subthalamic nucleus, substantia nigraC. Thalamus, subthalamic nucleus, zona incerta, substantia nigra D. Subthalamic nucleus, thalamus, zona incerta, substantia nigra

NeurosurgeryQ?

69

Page 62: Comprehensive Neurosurgery Board Review Flash cards

©™

NeurosurgeryA.• The correct answer is B, thalamus, zona

incerta, subthalamic nucleus, substantia nigra

• The thalamus (shown in green), subthalamic nucleus (in yellow) and the substantia nigra (in red) are also included in the basal ganglia. The putamen (in blue) can also be seen in this diagram. The zona incerta is the white area just inferior to the thalamus.

• The arrow depicts the trajectory traversed during DBS.

BOARD FAVORITE!

Fraix V, Pollak P, Van Blercom N, Xie J, Krack P, Koudsie A, Benabid AL (2000) Effect of subthalamic nucleus stimulation on levodopa-induced dyskinesia in Parkinson s disease. Neurology 55:1921–1923.

Classification: Neurosurgery, Deep Brain Stimulation, Structures

Page 63: Comprehensive Neurosurgery Board Review Flash cards

©™

• In doing deep brain stimulation of the subthalamic nucleus, going posterolateral will cause:A. Tetonic muscular contractionB. DysesthesiasC. Tremor reductionD. Double vision

NeurosurgeryQ?

70

Page 64: Comprehensive Neurosurgery Board Review Flash cards

©™

NeurosurgeryA.• The correct answer is B, dysesthesias.• NOTE –This is an important board question!

– Anterolateral: tetonic muscular contraction internal capsule– Dorsal: decrease in tremor zona incerta– Posterolateral: dysesthesias, sweating medial lemniscus– Medial: double vision, changes in pupils CN III, IV

BOARD FAVORITE!

Fraix V, Pollak P, Van Blercom N, Xie J, Krack P, Koudsie A, Benabid AL (2000) Effect of subthalamic nucleus stimulation on levodopa-induced dyskinesia in Parkinson s disease. Neurology 55:1921–1923.

Classification: Neurosurgery, Deep Brain Stimulation, Posterolateral to Subthalamic

Nucleus

Page 65: Comprehensive Neurosurgery Board Review Flash cards

©™

• A 76 year-old male presents to your clinic in consultation with results of a recent angiogram that suggest carotid stenosis with 45% occlusion by NASCET criteria. He denies current visual changes, headaches or weakness, but says that 2 weeks ago he did have “loss of sight” in the left eye for one day . You would recommend:

A. Medical therapy (ie, antiplatelet agents and cardiovascular risk factor control/prevention)

B. Recommend carotid endarterectomy after considering the patient's risk factor profile and comorbidities

C. Recommend endarterectomy without considering the patient's risk factor profile and comorbidities

D. Carotid endarterectomy plus antiplatelet therapy and cardiovascular risk factor control/prevention

NeurosurgeryQ?

98

Page 66: Comprehensive Neurosurgery Board Review Flash cards

©™

NeurosurgeryA.• The correct answer is A, medical therapy. (ie, antiplatelet agents and cardiovascular risk factor

control/prevention).• NASCET criteria for carotid stenosis:• Symptomatic (TIA or minor stroke) • Lower-grade carotid stenosis (<50% by NASCET criteria): medical therapy (ie, antiplatelet

agents and cardiovascular risk factor control/prevention)• Moderate-grade symptomatic carotid stenosis (50% to 69% by NASCET criteria): walk a finer

line. Consider patient's risk factor profile and comorbidities.• High-grade symptomatic carotid stenosis (70% to 99% by NASCET criteria): CEA plus

antiplatelet therapy and cardiovascular risk factor control/prevention is the treatment of choice.• Asymptomatic• >60% carotid stenosis (NASCET criteria): walk a fine line. If surgical complications exceed

3%, CEA would be harmful to the patient. Ferguson GG, Eliasziw M, Barr HWK, Clagett GP, et al. The North American Symptomatic Carotid Endarterectomy Trial. Stroke. 1999;30:1751-8.

BOARD FAVORITE!

Classification: Neurosurgery, NASCET, Carotid Stenosis

Page 67: Comprehensive Neurosurgery Board Review Flash cards

©™

Neurosurgery

• Match the appropriate indication for the given procedure:1. Dorsal root ganglionectomy 2. DREZ3. Cordotomy4. Midline myelotomy5. Intrathecal pump implant6. Spinal cord stimulator

A. Bilateral lesions, may cause Ondine’s CurseB. Severe spinal headache, seromaC. Unilateral allodyniaD. Incision 2 cm above pain siteE. Root avulsionF. Failed back syndrome; good for appendicular pain

Q?

103

Page 68: Comprehensive Neurosurgery Board Review Flash cards

©™

NeurosurgeryA.• The correct answer is….• To obtain the answer to this question and to view over 200 more

comprehensive neurosurgery questions please purchase the full product here!

H. Richard Winn, M.D. Youman’s Neurological Surgery 5th Edition. Philadelphia, PA: Elsevier 2004. p. 3028, 3059-61, 3112-4.Romanelli P, Esposito V, Adler J. Ablative procedures for chronic pain. Neurosurg Clin N Am. 2004 Jul;15(3):335-42. Review.

BOARD FAVORITE!

Classification: Neurosurgery, Surgical Procedures and Their Indications

Page 69: Comprehensive Neurosurgery Board Review Flash cards

©™©™

• A 47 year-old male presents with a complaint of pain “shooting down” his right leg. The MRI is shown here. Which of the following is the most appropriate management for this patient?

A. Do nothing, MRI looks normalB. Laminectomy and discectomyC. Laminectomy and instrumentation of

L5-S1D. Tethered cord release

NeuroradiologyQ?

2

Page 70: Comprehensive Neurosurgery Board Review Flash cards

©™©™

NeuroradiologyA.• The correct answer is B, laminectomy and

discectomy.• Lumbar spine MRI shows a right far-lateral

herniated disc (T2 axial view). Note that the disc bulge is not noted on the midline sagittal view.

• Far-lateral herniated discs commonly impinge on the exiting nerve root (same level) while more central discs affect the transiting nerve root (level below).

• These patients generally do well with simple laminectomy and discectomy. There is no need to perform laminectomy and instrumentation of L5-S1. There is no evidence of a thickened filum or low conus suggestive of a tethered cord.

H. Richard Winn, M.D. Youman’s Neurological Surgery 5th Edition. Philadelphia, PA: Elsevier 2004. p. 4514-7.

BOARD FAVORITE!

Classification: Neuroradiology, Herniated Disc, Laminectomy and Discectomy

Page 71: Comprehensive Neurosurgery Board Review Flash cards

©™©™

• This MRI of the brain is most suggestive of:A. Iron depositionB. MacrocraniaC. HypoxemiaD. Generalized atrophyE. Butterfly glioma

NeuroradiologyQ?

3

Page 72: Comprehensive Neurosurgery Board Review Flash cards

©™©™

NeuroradiologyA.• The correct answer is A, iron deposition.• Hallervorden-Spatz syndrome - involves the

deposition of iron especially in the globus pallidus and the retina.

• Dystonia, rigidity and neurobehavioral changes• MRI: Globus pallidus on T2-weighted images • "Eye-of-the-tiger" sign• Central region of hyperintensity

– Primary tissue insult – Produces edema

• Surrounding hypointensity – Region high in iron – May be 2° process

H. Richard Winn, M.D. Youman’s Neurological Surgery 5th Edition. Philadelphia, PA: Elsevier 2004. p. 2720.

BOARD FAVORITE!

Classification: Neuroradiology, Hallervorden Spatz Syndrome, Iron Deposition

Page 73: Comprehensive Neurosurgery Board Review Flash cards

©™©™

• Based on the AP and lateral cerebral angiogram of the right external carotid artery shown here, the most appropriate statement to tell this patient is that:

A. The risk of bleeding is high due to the cortical venous drainage.

B. The risk of bleeding is low due to the cortical venous drainage.

C. If the flow has rapid drainage into a sinus the bleeding risk is increased.

D. This is a normal cerebral angiogram.

NeuroradiologyQ?

4

Page 74: Comprehensive Neurosurgery Board Review Flash cards

©™©™

NeuroradiologyA.• The correct answer is A. The risk of bleeding

is high due to the cortical venous drainage.• This AP and lateral angiogram shows a dural

arteriovenous fistula with main feeders from the occipital and middle meningeal arteries. Multiple venous lakes and varices are identified including two large lesions (these suggest high pressure). Drainage of the dural AVF is via cortical veins primarily into the sagittal and transverse sinuses.

• The risk of bleeding is high not low with cortical venous drainage.

• If the flow has rapid drainage into a sinus the bleeding risk decreases.

Borden JA, Wu JK, Shucart WA. A proposed classification for spinal and cranial dural arteriovenous fistulous malformations and implications for treatment. J Neurosurg. 1995 Feb;82(2):166-79.

BOARD FAVORITE!

Occipital artery

Classification: Neuroradiology, duralarteriovenous fistula, cortical venous drainage

Page 75: Comprehensive Neurosurgery Board Review Flash cards

©™©™

• Locate the central sulcus:A. AB. BC. CD. DE. EF. F

NeuroradiologyQ?

ABCD

EF

8

Page 76: Comprehensive Neurosurgery Board Review Flash cards

©™©™

NeuroradiologyA.• The correct answer is C.• Sigmoid “Hook” or “Ducky Sign”• Hook-like configuration (B) of the posterior

surface of the precentral gyrus. The “hook” or “ducky’s breast” corresponds to the motor hand area. The “hook” is seen well on CT (89%) and MRI (98%).

• Pars bracket sign• The paired pars marginalis (F) form a

“bracket” to each side of the interhemispheric fissure at or behind the central sulcus (96%).

• A = precentral sulcus• D = postcentral gyrus• E = postcentral sulcus

Colen CB, Handbook of Neurosurgery and Neurology in Pediatrics, 2006

ABCD

EF

BOARD FAVORITE!

Classification: Neuroradiology, Neuroanatomy, Location of Central Sulcus

Page 77: Comprehensive Neurosurgery Board Review Flash cards

©™©™

• A 2 day old asymptomatic neonate is transferred to your institution with the following image. Which of the following is CORRECT regarding this pathology?A. It rarely presents with congestive heart

failure.B. It never presents in older children. C. The embryonic correlate to this

malformation is the median prosencephalic vein.

D. Treatment for this neonate should include immediate embolization.

NeuroradiologyQ?

11

Page 78: Comprehensive Neurosurgery Board Review Flash cards

©™©™

NeuroradiologyA.• The correct answer is C, the embryonic correlate to this

malformation is the median prosencephalic vein.• Vein of Galen aneurysmal malformations (VGAM) probably

represent an arteriovenous fistula (AVF) in the wall of a persistent embryonic vascular channel called the median prosencephalic vein.

• By week 10 of normal fetal development, the median prosencephalic vein regresses as the definitive internal cerebral veins appear. A caudal remnant remains as the vein of Galen.

• Neonatal presentation with congestive heart failure is frequent with these malformations.

Lasjuanias P. Vascular Diseases in Neonates, Infants and Children. New York: Springer Verlag 1997. Horowitz MB, Jungreis CA, Quisling RG, Pollock I. vein of Galen aneurysms: a review and current perspective. AJNR 1994; 15:1486-1496. H. Richard Winn, M.D. Youman’s Neurological Surgery 5th Edition. Philadelphia, PA: Elsevier 2004. p. 3434.

Vein of Galen aneurysmal malformation

•Initial treatment of VGAM is conservative. Embolisation of a neonate is a high risk procedure and the child should be treated medically (for cardiac failure) until aged 5 or 6 months with regular outpatient assessment. Elective embolisation can be scheduled for this time with the aim of closing the AVS with cyanoacrylate glue. If the infant deteriorates (seizures, failure to thrive, worsening cardiac failure, etc) treatment is performed earlier.

BOARD FAVORITE!

Classification: Neuroradiology, Vascular Pathology, Vein of Galen AneurysmalMalformations

Page 79: Comprehensive Neurosurgery Board Review Flash cards

©™©™

Neuroradiology

• A five-month-old male infant presents with an abnormal head shape. What is the most likely diagnosis?

A. Sagittal synostosisB. Bicoronal synostosisC. Left unicoronal synostosisD. Right unicoronal synostosisE. Metopic synostosis

Q?

12

Page 80: Comprehensive Neurosurgery Board Review Flash cards

©™©™

NeuroradiologyA.• The correct answer is C, left unicoronal synostosis.• Coronal synostosis can occur either on the right or left side

(unicoronal), or both (bicoronal.) It frequently occurs prenatally, and appears to occur more commonly in males. Restriction of normal cranial growth at one suture between the frontal and parietal bones produces a characteristic flattening of the forehead on the fused side, and a bulging of the forehead on the non-fused side.

• This bulging, or bossing, is a result of compensatory growth of the contralateral (opposite side) coronal suture. Secondary bulging of the temporal region on the ipsilateral (same) side can also occur. The placement of the eyes within the orbit is also typically affected. The superior margin of the orbit on the synostosed side is raised, a feature known as Harlequin eyedeformity.

H. Richard Winn, M.D. Youman’s Neurological Surgery 5th Edition. Philadelphia, PA: Elsevier 2004. p. 3300-01.

Contralateral•Frontal bossing•Displaced fontanelle

Ipsilateral•Flattened forehead•Raised brow(Harlequin Eye)•Temporal bulging

Classification: Neuroradiology, Infantile Pathology, Unicoronal Synostosis

Page 81: Comprehensive Neurosurgery Board Review Flash cards

©™©™Used with permission from Handbook of Neurosurgery andNeurology in Pediatrics; By Chaim B. Colen, MD, PhD.

• This x-ray demonstrates which of the following?A. McRae’s lineB. McGregor’s lineC. Rule of SpenceD. Chamberlain’s line

NeuroradiologyQ?

27

Page 82: Comprehensive Neurosurgery Board Review Flash cards

©™©™

NeuroradiologyA.• The correct answer is C, rule of Spence.• McRae’s line - foramen magnum diameter: 35 mm+/- 4• McGregor’s line - line from the hard palate to the caudal most part of the opisthion

(odontoid should be < 4 mm above this line > 4 mm suggests basilar impression). • Rule of Spence - if the sum of A + B ≥7mm, consider disruption of the transverse

ligament (requires rigid immobilization).• Chamberlain’s line - diagonal line from the hard palate to the posterior foramen

magnum (odontoid should not extend 1/3 of its height above this line).

Colen CB, Handbook of Neurosurgery and Neurology in Pediatrics, 2006.

BOARD FAVORITE!

Classification: Neuroradiology, Rule of Spence, Clinical Definition

Page 83: Comprehensive Neurosurgery Board Review Flash cards

©™©™

• What is the most likely diagnosis depicted in this MRI and MRS?A. Low grade gliomaB. Pleiomorphic xantho-

astrocytomaC. High grade gliomaD. None of the above

NeuroradiologyQ?

34

Page 84: Comprehensive Neurosurgery Board Review Flash cards

©™©™

NeuroradiologyA.• The correct answer is C, high grade glioma. • N-acetylaspartate (NAA) is predominantly located in neurons and is thus decreased in all

neoplasms that cause the neurons to be displaced or replaced with malignant cells. Findings of numerous studies have demonstrated decreased NAA values in glial neoplasms.

• Choline (Cho) peak contains contributions from glycerophosphocholine, phosphocholine, and phosphatidylcholine components that are thought to reflect cellular membrane density and turnover. As in any process that leads to hypercellularity and increased membrane proliferation, the Cho value is consistently elevated in gliomas.

• Lactate (Lac) indicates that cellular respiration has shifted from the oxidative metabolism of carbohydrates to nonoxidative metabolism. Increased reliance on anaerobic glycolysis is found in highly malignant tumors.

Law M, Hamburger M, Johnson G, Inglese M, Londono A, Golfinos J, Zagzag D, Knopp EA. Differentiating surgical from non-surgical lesions using perfusion MR imaging and proton MR spectroscopic imaging. Technol Cancer Res Treat. 2004 Dec;3(6):557-65. Review.

BOARD FAVORITE!

Classification: Neuroradiology, Magnetic Resonance Spectroscopy, High Grade Glioma

Page 85: Comprehensive Neurosurgery Board Review Flash cards

©™©™

• A 64 year-old black male presented with left sided headache and slurring of speech. Head CT is shown. Which of the following would likely be found on physical examination?A. Contralateral hemiparesisB. Ipsilateral hemiparesisC. Restricted left (lateral) gazeD. Restricted right (medial) gazeE. Locked-in-syndromeF. Both A and C

NeuroradiologyQ?

49

Page 86: Comprehensive Neurosurgery Board Review Flash cards

©™©™

NeuroradiologyA.• The correct answers are A, (C and F).• This patient has Millard Gubler syndrome!• REMEMBER - 7th nerve (7 letters in Millard) and - 6th nerve (6 letters in Gubler)• PLUS the corticospinal tract.• Millard-Gubler syndrome is associated with abducens (CN6) and facial nerve

(CN7) paralysis, as well as contralateral hemiplegia of the extremities. It involves unilateral damage to the inferior pons, commonly caused by pontine infarction or hemorrhage, and leads to damage of the above structures. The muscles of the ipsilateral side of the face are paralyzed, diplopia, internal strabismus, and loss of extroversion are also typically present.

Onbas O, Kantarci M, Alper F, Karaca L, Okur A. Millard-Gubler syndrome: MR findings. Neuroradiology. 2005 Jan;47(1):35-7.

Classification: Neuroradiology, Millard Gubler Syndrome, Physical Exam

Page 87: Comprehensive Neurosurgery Board Review Flash cards

©™©™

• A 57 year-old male underwent transnasal/ transfacial ethmoidectomy for metastatic squamous cell carcinoma. Post-operatively he developed clear rhinorrhea. Lumbar pucture was performed daily for 3 days draining 20 cc each time. On the third day he became confused and lethargic. On examination he was non-verbal and localized to painful stimulus. Head CT is shown. What is his most likely diagnosis?A. Recurrent tumorB. MeningitisC. Tension pneumocephalusD. Normal postoperative scan

NeuroradiologyQ?

50

Page 88: Comprehensive Neurosurgery Board Review Flash cards

©™©™

NeuroradiologyA.• The correct answer is C, tension pneumocephalus.• This head CT shows the Mt Fuji sign - twin peaks with bifrontal pneumocephalus.

The Mount Fuji sign is a finding that can be observed on computed tomographic (CT) scans of the brain (1), in which bilateral subdural hypoattenuating collections cause compression and separation of the frontal lobes. The collapsed frontal lobes and the widening of the interhemispheric space between the tips of the frontal lobes have the appearance of the silhouette of Mount Fuji hence, the Mount Fuji sign.

• Given this patient’s decline in mental status he is likely developing tension pneumocephalus.

H. Richard Winn, M.D. Youman’s Neurological Surgery 5th Edition. Philadelphia, PA: Elsevier 2004. p. 616.Ommaya AK: Cerebrospinal fluid fistula and pneumocephalus. In: Wilkins RH, Rengachary SS, eds. Neurosurgery. 2nd ed. New York: McGraw-Hill; 1996: 2773-82

Classification: Neuroradiology, Tension Pneumocephalus, CT scan

Page 89: Comprehensive Neurosurgery Board Review Flash cards

©™©™

• A 4 year-old boy is brought to the emergency room after a fall. Lateral skull x-ray is shown to the right. Which of the following is the most likely diagnosis?

A. EpidermoidB. Eosinophilic granulomaC. HemangioblastomaD. Normal skull x-ray

NeuroradiologyQ?

66

Page 90: Comprehensive Neurosurgery Board Review Flash cards

©™©™

NeuroradiologyA.• The correct answer is B, eosinophilic granuloma.• AP view of the skull reveals an approximate 3 cm. circular lytic lesion overlying the left

frontoparietal region with well delineated non-sclerotic margins, suggestive of histiocytosis X (eosinophilic granuloma).

• Differential Diagnosis: Solitary lytic lesion in the skull includes metastatic lesion, myeloma, fibrous dysplasia, tuberculosis, trauma, histiocytosis X, osteomyelitis, leptomeningeal cyst, and epidermoid/dermoid.

• Eosinophilic granuloma - lytic lesion with well delineated non-sclerotic margins and beveled edges.

• Epidermoids - lytic lesion with sclerotic margins.

Yang JT, Chang CN, Lui TN, Ho YS. Eosinophilic granuloma of the skull--report of four cases. Changgeng Yi Xue Za Zhi. Dec 1993;16(4):257-62.

BOARD FAVORITE!

Classification: Neuroradiology, Pediatric Pathology, Eosinophilic Granuloma

Page 91: Comprehensive Neurosurgery Board Review Flash cards

©™©™

• Treatment for this lesion in a patient with neurofibromatosis type -1 should consist of:A. Removing the optic nerve and

attached globe.B. Removing the optic nerve and both

globes.C. Radiotherapy with 25 GrayD. Radiotherapy with 80 Gray

NeuroradiologyQ?

67

Page 92: Comprehensive Neurosurgery Board Review Flash cards

©™©™

NeuroradiologyA.• The correct answer is A, removing the optic

nerve and attached globe.• MRI demonstrates a high intensity lesion of

the distal right optic nerve, suggestive of optic glioma, especially in a patient with neurofibromatosis type 1.

• Treatment of optic glioma distal to the chiasm involves removing the optic nerve and attached globe.

• Treatment of optic glioma involving the chiasm involves removing the optic nerve, globe and radiotherapy (less than 8 Gray). Board Favorite!

Hollander MD, FitzPatrick M, O'Connor SG, et al: Optic gliomas. Radiol Clin North Am 1999 Jan; 37(1): 59-71.

BOARD FAVORITE!

Classification: Neuroradiology, NF-1 MRI, Treatment Modalities

Page 93: Comprehensive Neurosurgery Board Review Flash cards

©™©™

• This tumor most likely has elevated serum levels of which of following:A. Alpha-fetoproteinB. Carcinoembryonic antigen (CEA)C. Cancer antigen-125 (CA-125)D. β-human chorionic gonadotropin (β-

HCG)E. Placental alkaline phosphatase (PLAP)

NeuroradiologyQ?

69

Page 94: Comprehensive Neurosurgery Board Review Flash cards

©™©™

NeuroradiologyA.• The correct answer is D, β-HCG.• T1W non-contrast MRI sagittal image shows a pineal

recess mass compressing the tectal plate and causing triventricular hydrocephalus.

• INTRACRANIAL GERM CELL TUMORS• Germinoma is the most common.• Most common tumor of pineal recess 40%• Males• Respond well to chemo and radiotherapy• Cancer antigen-125 (CA-125)- ovarian cancer• Placental alkaline phosphatase (PLAP) - ovarian

cancer

Colen CB, Handbook of Neurosurgery and Neurology in Pediatrics, 2006 p. 41

BOARD FAVORITE!

Classification: Neuroradiology, Intracranial Germ Tumors, Serum levels

Teilum Concept

Page 95: Comprehensive Neurosurgery Board Review Flash cards

©™©™

• A 20 year-old female presents with 3 weeks of headaches, nausea, and vomiting. MRI is shown here. Which of the following is the most appropriate next step in management?A. Operative resectionB. No intervention neededC. Shunt placement or third

ventriculostomyD. RadiotherapyE. A and C

NeuroradiologyQ?

70

Page 96: Comprehensive Neurosurgery Board Review Flash cards

©™©™

NeuroradiologyA.• The correct answer is….• To obtain the answer to this question and to view over 250 more comprehensive

neuroradiology questions please purchase the full product here !

Barkovich AJ, Krischer J, Kun LE, et al: Brain stem gliomas: a classification system based on magnetic resonance imaging. Pediatr Neurosurg 1990-91;16(2): 73-83

BOARD FAVORITE!

Classification: Neuroradiology, TectalGlioma, Treatment Modalities

Page 97: Comprehensive Neurosurgery Board Review Flash cards

©™©™

• This patient most likely sustained damage to which of the following structures?

A. Bilateral hypoglossal nucleiB. Left hypoglossal nerveC. Left hypoglossal nucleusD. Left upper motor neuron to the

hypoglossal nucleusE. Right vagal nucleus

NeurologyQ?

2

Page 98: Comprehensive Neurosurgery Board Review Flash cards

©™©™

NeurologyA.• The correct answer is D, left upper motor neuron to the hypoglossal nucleus.• Damage to the left hypoglossal nucleus (or nerve) would cause tongue

deviation to the left (ipsilateral). However, as depicted in the photo, damage to the upper motor neurons (which cross), would cause tongue deviation to the right (contralateral).

Midstokke S, Hess SJ, Saini T, Edwards PC. Unilateral tongue atrophy. Gen Dent. 2006 Nov-Dec;54(6):425-7.

BOARD FAVORITE!

Classification: Neurology, upper motor neuron lesion, hypoglossal nucleus

Page 99: Comprehensive Neurosurgery Board Review Flash cards

©™©™

• The function of the superior olive is:A. Sound localizationB. ProprioceptionC. Vestibular localization and functionD. Visual gaze control

NeurologyQ?

6

Page 100: Comprehensive Neurosurgery Board Review Flash cards

©™©™

NeurologyA.• The correct answer A, sound localization.• The superior olivary nucleus (or superior

olive) is a small mass of gray substance situated on the dorsal surface of the lateral part of the trapezoid body.

• The superior olivary nucleus’ primary input is bilateral and from the bushy cells of the anterior ventral cochlear nuclei (AVCN). This input occurs primarily via the ventral acoustic stria. Its output is to the lateral lemnisci.

• The superior olivary nucleus is the first point where binaural input is combined.

Kandel ER, Schwartz JH, Jessell TM. Principles of Neural Science, 4th ed. McGraw-Hill, New York. 2000. p. 606.

Superior Olivary nucleus

Cochlear nucleus

Superior and Inferior colliculi

Nucleus gracilis

Nucleus cuneatus

BOARD FAVORITE!

Classification: Neurology, Superior Olive Nucleus, Function

Page 101: Comprehensive Neurosurgery Board Review Flash cards

©™©™

• The function of the superior olive is demonstrated by which waveform:

A. 1B. 2C. 3D. 4E. 5F. 6G. 7

NeurologyQ?

76

54

321

7

Page 102: Comprehensive Neurosurgery Board Review Flash cards

©™©™

NeurologyA.• The correct answer C, 3.• Brainstem auditory evoked responses test the

integrity of the auditory pathway.• Wave 1 –auditory nerve• Wave 2 –Cochlear nuclei (pons)• Wave 3 –Superior olivary complex• Wave 4 –Lateral lemniscus• Wave 5 –Inferior colliculus• Wave 6 –Medial geniculate nucleus• Wave 7 –Auditory radiations (cortex)• Damage to one of the structures will result in

an increased latency! (e.g. acoustic neuroma will show increased latency of 1-3).

Kandel ER, Schwartz JH, Jessell TM. Principles of Neural Science, 4th ed. McGraw-Hill, New York. 2000. p. 606.

KNOW THESE WAVEFORMS WELL! BOARD FAVORITE!

BOARD FAVORITE!

76

54

321

Classification: Neurology, Evoked Potential, Superior Olive

Page 103: Comprehensive Neurosurgery Board Review Flash cards

©™©™

• Match the following statements with the correct answer:1. Lateral vestibulospinal tract2. Rubrospinal tract

A. Extensor toneB. Flexor toneC. BothD. Neither

NeurologyQ?

9

Page 104: Comprehensive Neurosurgery Board Review Flash cards

©™©™

NeurologyA.1. The correct answer is A, extensor tone. The lateral vestibulospinal tract regulates

extensor tone through the lateral vestibular nucleus.• The vestibulospinal tract arises from the lateral vestibular nucleus (i.e. Deiter’s

nucleus) and descends bilaterally in the anterior part of the lateral funiculus. 2. The correct answer is B, flexor tone. Flexor activity is regulated by the rubrospinal

tract from the red nucleus.• The rubrospinal tract arises from magnocellular neurons in the red nucleus and

crosses at the ventral tegmental decussation. Stimulation of the red nucleus leads to excitation of contralateral flexor alpha motor neurons and inhibition of extensor alpha motor neurons.

Kandel ER, Schwartz JH, Jessell TM. Principles of Neural Science, 4th ed. McGraw-Hill, New York. 2000. p. 668.

BOARD FAVORITE!

Classification: Neurology, Motor Tracts, lateral vestibulospinal and rubrospinal tract

Page 105: Comprehensive Neurosurgery Board Review Flash cards

©™©™

• Which brainstem nuclei release serotonin?A. Raphé nucleiB. Vestibular nucleiC. Hypoglossal nucleiD. Nucleus cuneatusE. Nucleus gracilis

NeurologyQ?

11

Page 106: Comprehensive Neurosurgery Board Review Flash cards

©™©™

NeurologyA.• The correct answer is A, raphé nuclei. The raphé nuclei are located in the brainstem.

Dorsal raphé nuclei

Dorsal raphé nuclei

Neurons in the dorsal raphé nucleus produce serotonin, they have long projections (green arrow) that carry the neurotransmitter to the orbital prefrontal cortex.

CROSS SECTION

Classification: Neurology, Brainstem Nuclei, Serotonin Releasing Cells

Page 107: Comprehensive Neurosurgery Board Review Flash cards

©™©™

• Lack of sensory loss, presence of a “pinch sign”, and hand weakness is characteristic of:

A. Carpal tunnel syndromeB. Anterior interosseous syndromeC. Ulnar nerve entrapmentD. Posterior interosseous syndrome

NeurologyQ?

32

Page 108: Comprehensive Neurosurgery Board Review Flash cards

©™©™

NeurologyA.• The correct answer is B, anterior interosseous syndrome. The “pinch sign” often occurs when there

is a complete anterior interosseous nerve (AIN) lesion. Attempts to pinch the tips of the terminal phalanges of the index finger and thumb results in an extension of the distal phalanges. Thus, the pulps rather than the tips of these two digits approximate.

Note the distinctive findings of a AIN syndrome on performing the pinch test. There is loss of function in the AIN-innervated flexor pollicis longus (FPL) and flexor digitorum profundus (FDP) muscles, leading to weakness in the distal phalanges of the thumb and forefinger. The left photograph shows the normal function when attempting this hand posture.

NORMAL PINCH SIGN

H. Richard Winn, M.D. Youman’s Neurological Surgery 5th Edition. Philadelphia, PA: Elsevier 2004. p. 3925..

BOARD FAVORITE!

Classification: Neurology, Diagnostic Tests, anterior interosseous syndrome

Page 109: Comprehensive Neurosurgery Board Review Flash cards

©™©™

• 30 year-old female presents with a history of shunted pseudotumor cerebri and small ventricles, now complains of worsening headache. Fundoscopic exam is shown below. Which of the following statements is TRUE?

A. It is best treated with Diamox.B. She likely has shunt failure and stiff ventricles.C. She has venous thrombosis.D. There is no cause for concern about this headache.

NeurologyQ?

63

Page 110: Comprehensive Neurosurgery Board Review Flash cards

©™©™

NeurologyA.• The correct answer is B, she likely has shunt failure and stiff ventricles. A low index of

suspicion should be present when evaluating a patient with shunted pseudotumor cerebri (PTC). Approximately 40% of shunts placed in children will fail in the 1st year and almost all children will require shunt revision at some point. In PTC, shunt malfunction may result in blindness.

• PTC is encountered most frequently in young, overweight women between the ages of 20 and 45. Headache, occurring in more than 90 percent of cases, is the most common presenting complaint. Dizziness, nausea, and vomiting may also be encountered, but there are typically no alterations of consciousness or higher cognitive function.

• It is defined clinically by four criteria: (1) elevated intracranial pressure, as demonstrated by lumbar puncture; (2) normal cerebral anatomy, as demonstrated by neuroradiographic evaluation; (3) normal cerebrospinal fluid composition; and (4) signs and symptoms of increased intracranial pressure, including papilledema.

Martin TJ, Corbett JJ: Pseudotumor cerebri, in Youmans JR(ed): Neurological Surgery, ed 4. Philadelphia: WB Saunders,1996, Vol 4, pp 2980–2997.

Classification: Neurology, pseudotumorcerebri, shunt failure

Page 111: Comprehensive Neurosurgery Board Review Flash cards

©™©™

NeurologyQ?• Based on the radiological features seen in this MRI,

choose the most appropriate statement regarding this condition:

A. They have no intervening brain tissue between the vascular spaces on histopathology.

B. They do have intervening brain tissue between the vascular spaces on histopathology.

C. There is no genetic association.D. The HOX gene is highly associated.

79

Page 112: Comprehensive Neurosurgery Board Review Flash cards

©™©™

NeurologyA.• The correct answer is A, they have no intervening brain tissue between the vascular spaces on

histopathology.• Cavernous malformations (i.e. angioma, cavernoma, or cavernous hemangioma) may be inherited or

sporadic and consist of variable sized sinusoids or cavernous spaces between capillaries. Unlike arteriovascular malformations and capillary telangiectasias., they have no intervening brain tissuebetween the vascular spaces and have thus been described as “blood sponges”. MRI shows well-defined, usually rounded lesions with little or no mass effect and are without vasogenic edema (unless hemorrhage is present). There may be small areas of new or old hemorrhages shown as a rim of hemosiderosis around the cavernous angioma in the surrounding brain tissue.

• Genetics: more common in Hispanics.• CCM1 (for cerebral cavernous malformation 1) -chromosome 7 at band 7q11.2-q21. It is also known as

KRIT1, for the protein created by the gene. 40% of familial cavernous angiomas. • CCM2 -band 7p15-p13, protein named malcavernin. 20% of familial cavernous angiomas. • CCM3 identified as linked to familial cavernous angioma is on chromosome 3 at band 3q.

Craig HD, Gunel M, Cepeda O, et al: Multilocus linkage identifies two new loci for a mendelian form of stroke, cerebral cavernous malformation, at 7p15-13 and 3q25.2-27. Hum Mol Genet 1998 Nov; 7(12): 1851-8.

BOARD FAVORITE!

Classification: Neurology, Cavernous Malformations, Histopathology

Page 113: Comprehensive Neurosurgery Board Review Flash cards

©™©™

NeurologyQ.• This fundus is MOST likely seen in which of the

following patients?A. 25 year-old femaleB. 14 year-old diabetic maleC. 75 year-old diabetic maleD. 25 year-old obese maleE. 25 year-old obese female

80

Page 114: Comprehensive Neurosurgery Board Review Flash cards

©™©™

NeurologyA.• The correct answer is E, 25 year-old obese female.• This ocular fundoscopic image demonstrates classic

grade 2 papilledema most likely seen in a 25 year-old obese female with pseudotumor cerebri (idiopathic intracranial hypertension).

Brazis PW, Lee AG: Elevated intracranial pressure and pseudotumor cerebri. Curr Opin Ophthalmol 1998 Dec; 9(6): 27-32.

Grade II papilledema. The halo of edema now surrounds the optic disc.

Classification: Neurology, PseudotumorCerebri, Papilledema

Page 115: Comprehensive Neurosurgery Board Review Flash cards

©™©™

• In myopathic disorders the motor unit potentials on electromyography would most likely show which of the following?

A. Increased amplitudeB. Decreased durationC. MonophasicD. Few in numbers with decreased recruitmentE. A and D

NeurologyQ?

86

Page 116: Comprehensive Neurosurgery Board Review Flash cards

©™©™

NeurologyA.• The correct answer is….• To obtain the answer to this question and to view over 300 more comprehensive

neurology questions please purchase the full app here !

Daube JR: The description of motor unit potentials in electromyography. Neurology 1978 Jul; 28(7): 623-5.

Low amplitude wave of myopathy

Polyphasic “giant wave” of reinnervation

BOARD FAVORITE!

Classification: Neurology, Electromyography, Myopathic Disorders

Page 117: Comprehensive Neurosurgery Board Review Flash cards

©™©™

Neurocritical CareQ.• What is the hallmark indicator for disseminated intravascular coagulation (DIC)?

A. Decreased fibrin-split productsB. Increased factor XC. Low plateletsD. Decreased d-dimer

5

Page 118: Comprehensive Neurosurgery Board Review Flash cards

©™©™

Neurocritical CareA.• The correct answer is C, low platelets. Coagulation factors are decreased. Fibrin-split

products and d-dimers increase.Inciting factor: Ex. crush injury, endotoxin

Systemic activation of coagulation factors

Widespread intravascularfibrin deposition

Consumption of plateletsand clotting factors

Thrombosis & organ failure Thrombosis & organ failure

DIC

Classification: Neurocritical Care, Coagulation Disorders, DIC

Page 119: Comprehensive Neurosurgery Board Review Flash cards

©™©™

• A 36 year old male presents to the emergency room as a trauma code. During resuscitation, clear fluid is noted dripping out of the left nare. An appropriate statement regarding the bedside glucose test is that it has:

A. high sensitivity, low specificityB. high sensitivity, high specificityC. low sensitivity, low specificityD. high sensitivity, low specificity

Neurocritical CareQ?

22

Page 120: Comprehensive Neurosurgery Board Review Flash cards

©™©™

Neurocritical CareA.• The correct answer is C, low sensitivity, low specificity. The bedside glucose test has low

sensitivity and low specificity. Therefore, it should be used only for trying to rule out a CSF leak rather than ruling it in.

• In the emergent setting, when evaluating a patient with rhinorrhea, if the rhinorrhoea contains glucose, the specificity of the test for CSF can be improved by excluding other factors that increase the glucose concentration of nasal discharge. If the nasal discharge is not blood stained, the blood glucose (measured at the same time as CSF glucose) is <6 mmol.L–1, and there are no other symptoms of upper respiratory tract infection, such as sneezing, nasal blockage, cough, sore throat, sputum, or purulent nasal discharge, then this increases the likelihood that the discharge contains CSF. If rhinorrhoea does not contain glucose, then either it does not contain CSF or CSF glucose concentrations are below the limit of detection by the sticks (false negative measurement).

• Beta-2 transferrin test is both highly sensitive and specific but would require a few days to process the Western blot at most institutions.

Chan DT, Poon WS, IP CP, Chiu PW, goh KY. How useful is glucose detection in diagnosing cerebrospinal fluid leak? The rational use of CT and Beta-2 transferrin assay in detection of cerebrospinal fluid fistula. Asian J Surg. 2004 Jan;27(1):39-42.

Classification: Neurocritical Care, CSF Leak, Bedside Glucose Test

Page 121: Comprehensive Neurosurgery Board Review Flash cards

©™©™

• Name three functions of platelets:A. Attach, agglutinate, agranulateB. Adhere, activate, avascularizeC. Adhere, aggregate, agranulateD. Apoptosis, aggregate, agranulateE. Acidification, activate, agranulate

Neurocritical CareQ?

34

Page 122: Comprehensive Neurosurgery Board Review Flash cards

©™©™

VasospasmVasospasm

Vessel InjuryVessel InjuryAdherenceAdherence

AggregationAggregation

AgranulationAgranulation

Vascular phaseVascular phase Platelet phasePlatelet phase

GpIIb/IIIa

vWf

fibrinogen

GPIb-IX-V complexGPIb receptor

Collagen

Platelet

Endothelial cell

GPIb-IX-V complex

PhospholipaseC activation

+

Release of platelet-activation factors

AdherenceAdherence1

+

2

3

Neurocritical CareA.• The correct answer is C, adhere, aggregate, and agranulate (degranulate) (Hint AAA).

Yee DL, Bergeron AL, Sun CW, Dong JF, Bray PF. Platelet hyperreactivity generalizes to multiple forms of stimulation.J Thromb Haemost. 2006 Sep;4(9):2043-50

•Platelets can be activated by binding to collagen, which is mediated directly through glycoprotein VI (GPVI) or indirectly via von Willebrand factor (VWF) binding to GPIb-V–IX. Signal transduction from any of these receptors leads to phospholipase C recruitment, which mediates calcium mobilization, platelet shape change, degranulation, and activation of GPIIb/IIIa to allow binding of fibrinogen and platelet-platelet interactions.

Classification: Neurocritical Care, Coagulation Cascade, Platelet Function

Page 123: Comprehensive Neurosurgery Board Review Flash cards

©™©™

1. Choose the most common inherited cause of post-operative bleeding?A. Bernard-Soulier diseaseB. Von Willebrand diseaseC. Hemophilia AD. Hemophilia BE. Leiden factor disease

2. Inheritance of this disease is mostly:A. Autosomal dominantB. Autosomal recessive

Neurocritical CareQ?

36

Page 124: Comprehensive Neurosurgery Board Review Flash cards

©™©™

Neurocritical CareA.1. The correct answer is B, von Willebrand disease.2. The correct answer is A, autosomal dominant.• Von Willebrand disease is the most common coagulation disorder. Inheritance is

autosomal dominant transmission in 90% of cases. Children born with the disease have either low levels (quantitative defect) of von Willebrand factor (vWF), or defective form (qualitative defect) of the same protein.

• Bernard-Soulier: defective or decreased expression of the glycoprotein Ib/IX/V complex on the surface of the platelets. This complex is the receptor for von Willebrand factor (vWF), and the result of decreased expression is deficient binding of vWF to the platelet membrane at sites of vascular injury, resulting in defective platelet adhesion.

• Factor V Leiden is the most common hereditary hypercoagulability disorder. In this disorder the Leiden variant of factor V, cannot be inactivated by activated protein C.

Franchini M. Advances in the diagnosis and management of von Willebrand disease. Hematology. 2006 Aug;11(4):219-25.

Classification: Neurocritical Care, Coagulation Disorders, Von Willebrand Disease

Page 125: Comprehensive Neurosurgery Board Review Flash cards

©™©™

• You are consulted on a 22 year-old female who is one day postpartum and has been admitted to the intensive care unit with widespread petechiae, purpura, and mucocutaneous bleeding. Head CT is shown on the right. Which of the following is INCORRECT regarding this condition?

A. Blood clotting mechanisms are activated throughout the body.

B. Thrombocytopenia may occur.C. Thrombocytosis is commonD. Fibrin-split products are increased.

Neurocritical CareQ?

38

Page 126: Comprehensive Neurosurgery Board Review Flash cards

©™©™

Neurocritical CareA.• The correct answer is C, thrombocytosis is common• Thrombocytopenia occurs, not thrombocytosis.• The immediate postpartum period is a highly thrombogenic state. There are many risk factors

(amniotic fluid) that can cause a dysregulated hypercoagulable condition resulting in disseminated intravascular coagulation (DIC). DIC occurs when blood clotting mechanisms are activated throughout the body. Small blood clots form throughout the body, exhausting the supply of blood clotting factors which become rare at sites of real tissue injury. Simultaneously, clot dissolving mechanisms are also increased. Hence, this disorder can result in both excessive bleeding and clotting. DIC may be caused by infections (especially gram-negative bacteria) , severe trauma, cancer, blood transfusions, and obstetrical complications.

• Pathophysiology: two main mechanisms- generation of thrombin (microvascular thrombosis and organ ischemia) and generation of plasmin (characterized mainly by hemorrhagic symptoms).

• Fibrin-split products are increased in DIC.

Levi M, de Jonge E, van der Poll T, ten Cate H. Advances in the understanding of the pathogenetic pathways of disseminated intravascular coagulation result in more insight in the clinical picture and better management strategies. Semin Thromb Hemost. 2001 Dec;27(6):569-75. Review.

Classification: Neurocritical Care, Coagulation Disorders, Post Partum

Page 127: Comprehensive Neurosurgery Board Review Flash cards

©™©™

Neurocritical CareQ?• This ocular fundus is MOST likely seen in which

of the following patients?A. 25 year-old obese female punched in the

stomach with a GCS of 15B. 14 year-old male status post temporal

lobectomyC. 31 year-old male thrown from a 3rd story

window with a GCS of 5D. 56 year-old with a 1 cm cerebellar

hematoma with a GCS of 15

46

Page 128: Comprehensive Neurosurgery Board Review Flash cards

©™©™

Neurocritical CareA.• The correct answer is C, 31 year-old male thrown from a

3rd story window with a GCS of 5.• This type of papilledema is seen in the setting of high

intracranial pressure (ICP), such as the above patient who has suffered severe traumatic brain injury.

• Cerebellar hematoma acutely would cause high ICPs in the infratentorial compartment that potentially could result in acute obstructive hydrocephalus with papilledema, but this is less likely in a patient that has a 1 cm cerebellar hematoma and a GCS of 15.

Bhatt UK. Bilateral optic disc swelling; is a CT scan necessary? Emerg Med J. 2005 Nov;22(11):827-30.

Grade IV papilledema. There is severe swelling in addition to a circumferential halo. The edema covers major blood vessels as they leave the optic disk (grade III) and vessels on the disk (grade IV).A subretinal hemorrhage is present at 7 o'clock. 

Classification: Neurocritical Care, Hematoma, Ocular Changes

Page 129: Comprehensive Neurosurgery Board Review Flash cards

©™©™

• Which of the following is the preferred ventilation pattern in non-paralyzed patients with central nervous system dysfunction?

A. Continuous mandatory ventilationB. Assist control ventilationC. Intermittent mandatory ventilationD. Synchronous intermittent mandatory ventilationE. Pressure support ventilation

Neurocritical CareQ?

47

Page 130: Comprehensive Neurosurgery Board Review Flash cards

©™©™

Neurocritical CareA.• The correct answer is D, synchronous intermittent mandatory ventilation.• Patients with CNS dysfunction tend to exhibit variations in tidal volume and respiratory

drive. • Synchronized intermittent mandatory ventilation (SIMV) delivers volume-cycled

breaths that coincide with spontaneous lung inflations. The patient can breathe unassisted and spontaneously between mechanical breaths. Also, a preselected respiratory rate can be used if the patient`s respiratory drive is insufficient to prevent hypercarbia.

Stock MC, Perel A: Handbook of mechanical ventilatory support. Baltimore: Williams & Wilkins; 1997.

Classification: Neurocritical Care, CNS dysfunction, breathing patterns

Page 131: Comprehensive Neurosurgery Board Review Flash cards

©™©™

• Which of the following characteristics are seen in patients with acute respiratory distress syndrome (ARDS)?

A. Bilateral lung infiltratesB. Pulmonary capillary wedge pressure less than or equal to 18 mm Hg C. PAO2/FIO2 ratio of < 200 mm HgD. All of the above

Neurocritical CareQ?

53

Page 132: Comprehensive Neurosurgery Board Review Flash cards

©™©™

Neurocritical CareA.• The correct answer is D, all of the above.• Acute respiratory distress syndrome (ARDS) is characterized by acute onset, bilateral

infiltrates in chest x-ray, pulmonary capillary wedge pressure (PCWP) less than or equal to 18 mm Hg and a PAO2/FIO2 ratio < 200 mm Hg.

Kollef MH, Schuster DP: The acute respiratory distress syndrome. N Engl J Med 1995 Jan 5; 332(1): 27-37.

Classification: Neurocritical Care, Respiratory Distress Syndrome, Lab Values

BOARD FAVORITE!

Page 133: Comprehensive Neurosurgery Board Review Flash cards

©™©™

• Which of the following findings will distinguish cardiogenic pulmonary edema from acute respiratory distress syndrome (ARDS)?

A. Bilateral lung infiltrates on chest x-rayB. HypoxemiaC. Pulmonary capillary wedge pressure (PCWP) > 18 mm HgD. All of the above

Neurocritical CareQ?

55

Page 134: Comprehensive Neurosurgery Board Review Flash cards

©™©™

Neurocritical CareA.• The correct answer is C, Pulmonary capillary wedge pressure (PCWP) > 18 mm Hg.• Occasionally it is very difficult to distinguish between ARDS and cardiogenic

pulmonary edema. Both can present with hypoxia and bilateral lung infiltrate, but PCWP is > 18 mm Hg in cardiogenic pulmonary edema.

Ware LB, Matthay MA: The acute respiratory distress syndrome. N Engl J Med 2000 May 4; 342(18): 1334-49.

Classification: Neurocritical Care, Acute Respiratory Disress Syndrome, Pulmonary Changes

BOARD FAVORITE!

Page 135: Comprehensive Neurosurgery Board Review Flash cards

©™©™

• What is most common finding on ECG in patients with pulmonary emboli?A. Large Q wave in lead IIIB. Inverted T wave in lead III C. Right axis deviationD. Wide S complex in lead IE. Right bundle branch blockF. Sinus tachycardia

Neurocritical CareQ?

63

Page 136: Comprehensive Neurosurgery Board Review Flash cards

©™©™

Neurocritical CareA.• The correct answer is F, sinus tachycardia.• All of the answers can be seen in case of pulmonary emboli. The classic findings of

“S1,Q3,T3” are not very sensitive in diagnosis of acute pulmonary embolism.

Feied CF: Pulmonary embolism. In: Rosen and Barkin, eds. Emergency Medicine Principles and Practice. Vol 3. 4th ed. 1998:chap 111.

Classification: Neurocritical Care, Pulmonary Emboli, ECG Changes

BOARD FAVORITE!

Page 137: Comprehensive Neurosurgery Board Review Flash cards

©™©™

• A 21 year-old patient with history of recurrent sinusitis developed an allergic reaction after transfusion of a cross matched blood. What is the most likely cause?

A. Selective IgA deficiencyB. Lab errorC. Rh group incompatibilityD. Rapid transfusion of blood products

Neurocritical CareQ?

66

Page 138: Comprehensive Neurosurgery Board Review Flash cards

©™©™

Neurocritical CareA.• The correct answer is A, selective IgA deficiency.• Selective IgA deficiency syndrome is the most common primary immunodeficiency.

Usually the patient is asymptomatic, but a history of recurrent infections may be present; most commonly ear infections, sinusitis, and pneumonia. Allergies are another possible symptom including allergic reaction to blood products.

• If only fever develops within 1-6 hours of receiving cross matched blood, this is caused by antibodies in recipient blood that react to donor leukocytes. Leukocyte poor red cells can be used to prevent this reaction.

Ballow M. Primary immunodeficiency disorders: antibody deficiency. J Allergy Clin Immunol. Apr 2002;109(4):581-91.Zhao SM. Clinical assessment of preventing febrile nonhemolytic transfusion reaction by leukocyte-depleted blood transfusion. 2002 Dec;10(6):568-70.

Classification: Neurocritical Care, Congenital Immune Disorders, IgA Def. Syndrome

BOARD FAVORITE!

Page 139: Comprehensive Neurosurgery Board Review Flash cards

©™©™

• Which of the following is the appropriate immediate treatment in case of an intra-operative venous air embolism?

A. Irrigation of the surgical fieldB. Lower the patient’s head with lateral left decubitus positionC. Aspiration of air from a multiforce central venous pressure catheterD. Manual occlusion of the jugular veinsE. All of the above

Neurocritical CareQ?

70

Page 140: Comprehensive Neurosurgery Board Review Flash cards

©™©™

Neurocritical CareA.• The correct answer is E, all of the above.• Intraoperative venous air embolism (VAE) has a high incidence during procedures

performed in the sitting position. VAE is characterized by the development of bronchoconstriction, hypoxia, hypercarbia, hypotension, shock, cardiac arrhythmias, increased airway pressure, and decreased end tidal CO2.

• The most sensitive diagnostic modality for VAE is transesophageal echocardiography.

Ballki M.Venous air embolism during awake craniotomy in a supine patient.Can J Anaesth. 2003 Oct;50(8):835-8.

Classification: Neurocritical Care, Intraoperative Venous Embolism, Treatment

BOARD FAVORITE!

Page 141: Comprehensive Neurosurgery Board Review Flash cards

©™©™

• Match the following arterial blood gas values with the appropriate acid/base disorder. A. Metabolic acidosisB. Metabolic alkalosisC. Respiratory acidosisD. Respiratory alkalosisE. Combined respiratory acidosis and metabolic acidosis

1. pH 7.26, PaCO2 53 mmHg, HCO3- 24 mEq/L

2. pH 7.49, PaCO2 46 mmHg, HCO3- 33 mEq/L

3. pH 7.30, PaCO2 33 mmHg, HCO3- 18 mEq/L

4. pH 7.27, PaCO2 41 mmHg, HCO3- 18 mEq/L

Neurocritical CareQ?

75

Page 142: Comprehensive Neurosurgery Board Review Flash cards

©™©™

Neurocritical CareA.• The correct answers are….• To obtain the answer to this question and to view over 250 more comprehensive

neurocritical care questions please purchase the full product here !

Narins RG, Emmett M: Simple and mixed acid-base disorders: a practical approach. Medicine (Baltimore) 1980; 59(3): 161-87.

Classification: Neurocritical Care, Acid Base Disorders, Lab Values

BOARD FAVORITE!

Page 143: Comprehensive Neurosurgery Board Review Flash cards

©™

• Match the following statements with the correct answer:

1. Neuromuscular junction2. Botox3. Renshaw cell4. Blocks Renshaw cell

neurotransmitter5. Curare

NeurobiologyQ?

A. Acetylcholine releaseB. Acetylcholine receptorC. 5 subunits: 2α subunits, a β, a δ and either

a γ or an ε.D. StrychnineE. Glycine

4

Page 144: Comprehensive Neurosurgery Board Review Flash cards

©™

NeurobiologyA.• The corrects answers are the following: 1 - C, 2 - A, 3 - E, 4 - D, 5 - B. • Neuromuscular junction- the neuromuscular junction contains the nicotinic receptor

(an acetylcholine receptor subtype (nAChR)) There are 2 types of nAChRs; autonomic and neuromuscular. The autonomic nicotinic receptor has 5 subunits: 3α and 2β. The neuromuscular nAChR also consists of 5 subunits:2α subunits, a β, a δ and either a γ or an ε subunit.

• Botox –botulism toxin decreases Ach release (at neuromuscular junction and autonomic nerve terminals: both sympathetic & parasympathetic) by binding to synaptotagmin, while curare (δ-tubocurarine) blocks the Ach receptor. Botox has actually become useful for cosmetic purposes and dystonias.

• The Renshaw cell neurotransmitter is glycine; an aminoacid and whose channel is blocked by strychnine.

Dressler D, AdibSaberi F. New formulation of Botox: complete antibody-induced treatment failure in cervical dystonia. J NeurolNeurosurg Psychiatry. 2007 Jan;78(1):108-9Adams, M.E. and Olivera, B.M. Neurotoxins: overview of an emerging research technology. Trends Neurosci. 17:151-155. (1994).

BOARD FAVORITE!

Classification: Neurobiology, Neuromuscular junction, Receptor actions

Page 145: Comprehensive Neurosurgery Board Review Flash cards

©™

• Which of the following is the principle visceral nucleus of the brainstem?A. Nucleus ambiguusB. Raphe nucleusC. Nucleus solitariusD. Locus ceruleusE. Trigeminal nucleus

NeurobiologyQ?

5

Page 146: Comprehensive Neurosurgery Board Review Flash cards

©™

NeurobiologyA.• The correct answer is C, nucleus solitarius.• Nucleus solitarius receives afferent visceral information from the heart, lungs, GI

tract, as well as for taste. The solitary nucleus and tract are neural structures in the brainstem that carry and receive visceral sensation and taste from the facial (VII), glossopharyngeal (IX), vagus (X) cranial nerves, as well as the cranial part of the accessory nerve (XI).

• Nucleus ambiguous gives rise to the efferent motor fibers of the vagus nerve (CN X) terminating in the laryngeal and pharyngeal muscles, efferent motor fibers of the glossopharyngeal nerve (CN IX) terminating in the stylopharyngeus and contains the preganglionic parasympathetic neurons for cardiac regulation.

Andresen MC, Doyle MW, Bailey TW, Jin YH. Differentiation of autonomic reflex control begins with cellular mechanisms at the first synapse within the nucleus tractus solitarius. Braz J Med Biol Res. 2004 Apr;37(4):549-58.

BOARD FAVORITE!

Classification: Neurobiology, Brainstem, Nucleus Solitarius

Page 147: Comprehensive Neurosurgery Board Review Flash cards

©™

• Choose the MOST accurate statement regarding the physiology of an action potential:A. Voltage-gated potassium channels (also called delayed rectifier potassium

channels) have a delayed response, such that potassium continues to flow out of the cell and initiate depolarization.

B. Hyperpolarization is caused by K+ influx into a cell.C. Closing of voltage-gated potassium channels is both voltage- and time-

dependent.D. Depolarization is caused by Na+ efflux from a cell.

NeurobiologyQ?

35

Page 148: Comprehensive Neurosurgery Board Review Flash cards

©™

NeurobiologyA.• The correct answer is C, closing of voltage-gated potassium channels is both voltage- and time-

dependent. As potassium exits the cell, the resulting membrane repolarization initiates the closing of voltage-gated potassium channels.

Hyperpolarization

Resting potential

0 1 2 3 4 5Time (ms)

Threshold

Stimulus

Peak

Mem

bran

e vo

ltage

(mV

)

40

0

-55

-70

Na+ influx

K+ efflux

• These channels do not close immediately in response to a change in membrane potential. Rather, voltage-gated potassium channels (also called delayed rectifier potassium channels) have a delayed response, such that potassium continues to flow out of the cell even after the membrane has fully repolarized. Thus the membrane potential dips below the normal resting membrane potential of the cell for a brief moment; this dip of hyperpolarization is known as the undershoot.

• Hyperpolarization is caused by K+ effluxfrom a cell.

• Depolarization is caused by Na+ influx into a cell.

Kandel ER, Schwartz JH, Jessell TM. Principles of Neural Science, 4th ed. McGraw-Hill, New York. 2000. p. 128-33.Robbins and Cotran. Pathologic Basis of Disease, 6th Edition. W.B. Saunders Company. 1999. p.55-56.

BOARD FAVORITE!

Classification: Neurobiology, Neurophysiology, Action Potential

Page 149: Comprehensive Neurosurgery Board Review Flash cards

©™

• Neurotransmitter that is found in a postganglionic sympathetic nerve terminal:A. GABAB. SerotoninC. AcetylcholineD. NorepinephrineE. C + D

NeurobiologyQ?

36

Page 150: Comprehensive Neurosurgery Board Review Flash cards

©™

NeurobiologyA.• The correct answer is E, acetylcholine (Ach) and norepinephrine (NE).• Both Ach and NE are found in postganglionic sympathetic fibers. • Ach- sweat, piloerectors, and vasculature, mediated by muscarinic receptors

(inhibited by pertussis toxin).• REMEMBER: sweating of the palms of the hands is parasympathetic NOT

sympathetic! (BOARD FAVORITE!)• NE- all others• Peripheral sympathetic pathway –TRAVELS WITH SKELETAL NERVES (sweat,

piloerectors, and vasculature) (BOARD FAVORITE!)• Head and Neck sympathetic pathway -TRAVELS WITH BLOOD VESSELS

(BOARD FAVORITE!)• All preganglionic fibers (both sympathetic and parasympathetic) use Ach.

Kandel ER, Schwartz JH, Jessell TM. Principles of Neural Science, 4th ed. McGraw-Hill, New York. 2000. p. 1051-52.

BOARD FAVORITE!

Classification: Neurobiology, Neurophysiology, Sympathetic Nerve Terminal

Page 151: Comprehensive Neurosurgery Board Review Flash cards

©™

• Which of the following statements is true regarding the intracellular second messenger cyclic AMP.

A. It is involved with photoreceptionB. It is hydrolyzed by phospholipaseC. It synergistically activates protein kinase CD. It is increased with D1 receptor stimulation

NeurobiologyQ?

37

Page 152: Comprehensive Neurosurgery Board Review Flash cards

©™

NeurobiologyA.• The correct answer is D, it is increased with D1 receptor stimulation.• Hormones that utilize this system include: epinephrine and norepinephrine, glucagon

and TSH.• In a cAMP dependent pathway, the activated Gs alpha subunit binds to and activates

an enzyme called adenylyl cyclase which in turn catalyzes the conversion of ATP into cyclic adenosine monophosphate (cAMP). The intracellular second messenger cAMP activates protein kinase A (PKA).

• cAMP is decomposed to AMP by the action of the enzyme phosphodiesterase. This cAMP phosphodiesterase - has an activity opposite to kinase, therefore it dephosphorylates cAMP into AMP, reducing the cAMP levels.

• Deregulation of cAMP pathways (hyperactive) and an aberrant activation of cAMP-controlled genes is linked to the growth of some cancers

• Activation of conventional protein kinase C - requires DAG, Ca2+, and phospholipid.

Kandel ER, Schwartz JH, Jessell TM. Principles of Neural Science, 4th ed. McGraw-Hill, New York. 2000. p. 230-36.

BOARD FAVORITE!

Classification: Neurobiology, Neurophysiology, Second Messenger Systems

Page 153: Comprehensive Neurosurgery Board Review Flash cards

©™

• Match the receptors with their action:1. Mu12. Mu23. Kappa4. Delta5. Sigma

NeurobiologyQ?

A. HallucinationsB. Opiate analgesic euphoriaC. Miosis D. Reinforcing behaviorE. Respiratory depression

38

Page 154: Comprehensive Neurosurgery Board Review Flash cards

©™

NeurobiologyA.• The correct answers are:

1. B, Mu1- Opiate analgesic euphoria2. E, Mu2- Respiratory depression3. C, Kappa- Miosis4. D, Delta- Reinforcing behavior5. A, Sigma- Hallucinations

Kandel ER, Schwartz JH, Jessell TM. Principles of Neural Science, 4th ed. McGraw-Hill, New York. 2000. p. 483-6.Robbins and Cotran. Pathologic Basis of Disease, 6th Edition. W.B. Saunders Company. 1999. p.55-56.

BOARD FAVORITE!

Classification: Neurobiology, Pharmacology, Receptor Actions

Page 155: Comprehensive Neurosurgery Board Review Flash cards

©™

• Hexamethonium blocks which receptor?A. MuscarinicB. NicotinicC. Glycine D. Glutamate

NeurobiologyQ?

39

Page 156: Comprehensive Neurosurgery Board Review Flash cards

©™

NeurobiologyA.• The correct answer is B, nicotinic receptor.• Hexamethonium is a nicotinic Ach receptor antagonist, acts by preventing

acetylcholine from binding to the cholinergic receptor.• It has no effect on the muscarinic acetylcholine receptors, or mAChRs.

Kandel ER, Schwartz JH, Jessell TM. Principles of Neural Science, 4th ed. McGraw-Hill, New York. 2000. p. 971.

BOARD FAVORITE!

Classification: Neurobiology, Pharmacology, Receptor Actions

Page 157: Comprehensive Neurosurgery Board Review Flash cards

©™

• A 37 year-old male presented with generalized seizure and expressive dysphasia. MRI is shown. Reoccurrence of this tumor as a high-grade is LEAST likely due to which genetic abnormality?

NeurobiologyQ?

A. P53 tumor suppressorB. Retinoblastoma geneC. Amplification of platelet

derived growth factorD. Amplification of epidermal

growth factor receptor

41

Page 158: Comprehensive Neurosurgery Board Review Flash cards

©™

NeurobiologyA.• The correct answer is D, amplification of epidermal growth factor receptor.• This MRI demonstrates a low-grade glioma. Recurrent progressive low-grade glioma is

associated with mutations in the tumor suppressor gene p53 with amplification of CDK4 or loss of Rb and amplification or overexpression of PDGF.

• There are 2 subtypes of glioblastoma– Older patients with de novo tumors

• Deletions in cell cycle related INK4a-ARF genes p16INK4A and p19/p14ARF• EGFR amplifications

– Younger patients with progressive low grade glioma• Mutations in the tumor suppressor gene p53 with amplification of CDK4 or loss of

Rb• Amplification or overexpression of PDGF

Berger MS, Pardos M,Textbook of Neuro-Oncology. Saunders, Philadelphia. 2005. p. 608-9.

BOARD FAVORITE!

Classification: Neurobiology, Genetics, Tumor Growth Factors

Page 159: Comprehensive Neurosurgery Board Review Flash cards

©™

• Match the following sensory receptors with their description. 1. Meissner capsule2. Merkel receptors3. Free nerve endings4. Ruffini corpuscles5. Pacinian corpuscles

NeurobiologyQ?

A. Rapidly adaptive, vibration, receptors B. Slowly adaptive, touch and pressure, receptorC. Slowly adaptive, heavy pressure, receptorD. Rapidly adaptive, touch, receptors E. Pain receptors

60

Page 160: Comprehensive Neurosurgery Board Review Flash cards

©™

NeurobiologyA.• The correct answers are….• To obtain the answer to this question and to view over 250 more comprehensive neurobiology

questions please purchase the full product here !

Guyton, Arthur C., John E. Hall. Textbook of medical physiology. W.B. Saunders Company; 10th edition, 2000. p. 521, 530-2

Ruffini and Merkel drink TONIC SLOWLY! (slowly adapting tonic receptors)

R M

BOARD FAVORITE!

Classification: Neurobiology, Neurophysiology, Touch Receptors

Page 161: Comprehensive Neurosurgery Board Review Flash cards

©™©™

• Which of the following statements is INCORRECT regarding the anatomy of the cavernous carotid artery?

A. The inferior hypophyseal artery is most commonly a branch of the meningohypophyseal trunk.

B. The inferior hypophyseal artery passes medially to the posterior pituitary capsule.C. Persistent trigeminal arteries can also originate from the posterior vertical

segment of the cavernous internal carotid artery and pass posteriorly through the posterior wall of the cavernous sinus to join the basilar artery between the origin of the superior and anterior inferior cerebellar arteries.

D. The tentorial artery, or the artery of Bernasconi and Cassinari, is the most inconstant branch of the meningohypophyseal trunk.

NeuroanatomyQ?

1

Page 162: Comprehensive Neurosurgery Board Review Flash cards

©™©™

NeuroanatomyA.• The correct answer is D. The

meningohypophyseal trunk is the most inconsistent branch of the meningohypophyseal trunk.

• The meningohypophyseal trunk is the most constant artery (labeled below).

• The meningohypophyseal trunk usually arises from the posterior aspect of the central third of the posterior bend of the artery at the level of the dorsum sella and frequently gives rise to three branches: the tentorial artery (Bernasconi and Cassinari), the dorsal meningeal artery, and the inferior hypophyseal artery.

• All other answers are true statements.Isolan G, de Oliveira E, Mattos JP. Microsurgical anatomy of the arterial compartment of the cavernous sinus: analysis of 24 cavernous sinus. ArqNeuropsiquiatr. 2005 Jun;63(2A):259-64

V2

V1V1

Middle Fossa

CN5

CN4

Pituitary

CN3

CerebralPeduncle

Carotid

Meningohypophyseal trunk 

Tentorial artery

Inferior hypophyseal artery

Dorsal meningeal artery

Comments:  Cerebral Arteries, meningohypophyseal, Bernasconi and Cassinari

Page 163: Comprehensive Neurosurgery Board Review Flash cards

©™©™

• α-motor neurons are most commonly found in which Rexed lamina? A. IIIB. VC. VID. IXE. X

NeuroanatomyQ?

2

Page 164: Comprehensive Neurosurgery Board Review Flash cards

©™©™

NeuroanatomyA.• The correct answer is D, IX.• Rexed lamina IX, is located in the ventral motor

areas of the gray matter. Described in 1950s by Bror Rexed as a way to name the different parts of the spinal cord, the Rexed laminae consist of ten layers of grey matter (I-X). It must be noted that Rexed areas are not designated in terms of location, but rather by their cellular structure.

• Layer 1 – marginal zone, fast pain (Aδ) and temperature fibers

• Layer 2 – substantia gelatinosa,slow pain (C fibers)• Layer 4 – nucleus propius, interneurons to thalamus• Layer 7 – contains the zona intermedia• Layer 9 – α and γ motorneurons• Layers are all BOARD FAVORITE topics !Anamizu Y, Seichi A, Tsuzuki N, Nakamura K. Age-related changes in histogram pattern of anterior horn cells in human cervical spinal cord.Neuropathology. 2006 Dec;26(6):533-9.

7

98

3

1

9

10 9

2

6

45

BOARD FAVORITE!

Comments:  Spine, Rexed lamina

Page 165: Comprehensive Neurosurgery Board Review Flash cards

©™©™

• Match the following cortical neuroanatomy:

1. Sylvian fissure2. Rolandic fissure3. Pars triangularis4. Pars opercularis5. Pars orbitalis6. Supramarginal gyrus7. Angular gyrus

NeuroanatomyQ?

AB C

F

E

D

GH

36

Page 166: Comprehensive Neurosurgery Board Review Flash cards

©™©™

NeuroanatomyA.• Cortical neuroanatomy:

1. D, Sylvian fissure2. E, Rolandic fissure3. B, Pars triangularis4. C, Pars opercularis5. A, Pars orbitalis6. G, Supramarginal gyrus7. H, Angular gyrus

• Note: F is the precentral sulcus.

Motor cortex Sensory cortex

AB C

F

E

D

GH

H. Richard Winn, M.D. Youman’s Neurological Surgery 5th Edition. Philadelphia, PA: Elsevier 2004. p. 2533-7.

BOARD FAVORITE!

Comments:  Cortical anatomy, fissures, gyri

Page 167: Comprehensive Neurosurgery Board Review Flash cards

©™©™

• Match the following boundaries of Kawase’s triangle to their anatomic structure. A. Anteromedial boundaryB. Anterolateral boundaryC. Posterior boundary

1. Greater superficial petrosal nerve (GSPN)2. Internal auditory canal3. Lateral edge of the trigeminal nerve

NeuroanatomyQ?

37

Page 168: Comprehensive Neurosurgery Board Review Flash cards

©™©™

• The correct answers are A - 3, B - 1, C - 2.

• The anteromedial boundary is formed by the lateral edge of the trigeminal nerve.

• The anterolateral boundary is formed by the greater superficial petrosal nerve (GSPN).

• The posterior boundary is formed by the internal auditory canal or a line connecting the hiatus fallopii to the dural ostium of Meckel’s cave.

H. Richard Winn, M.D. Youman’s Neurological Surgery 5th Edition. Philadelphia, PA: Elsevier 2004. p. 921-5.

NeuroanatomyA.

Comments:  Kawase’s triangle, boundaries, CN V, Greater superficial petrosal nerve

Page 169: Comprehensive Neurosurgery Board Review Flash cards

©™©™

NeuroanatomyQ?• The vertebral arteries enter within the transverse foramina of which vertebrae:

A. C4 to C1B. C6 to C1C. C6 to C2D. C7 to C2E. C8 to C2

52

Page 170: Comprehensive Neurosurgery Board Review Flash cards

©™©™

NeuroanatomyA. BOARD FAVORITE!

Comments:  Extracranial Vasculature, Vertebral artery, Cervical spine, transverse foramina 

• The correct answer is B, C6 to C1.• The vertebral artery passes through the transverse

foramina of C6 through C1 before exiting the transverse foramina of the atlas (C1).

• At this location the vertebral artery curves posterior and superior to lie in a groove on the upper surface of the atlas .

• At C7 the vertebral artery swings anteriorly and laterally because it normally runs outside the rudimentary transverse foramina of C7. When a foramina is present, transverse foramina of the seventh cervical vertebra almost always contains vascular and nerve branches as well as fibrous and adipose tissues.

Katsuta T, Rhoton AL Jr, Matsushima T.The jugular foramen: microsurgical anatomy and operative approaches. Neurosurgery. 1997 Jul;41(1):149-201. MS Jovanovic. A comparative study of the foramen transversarium of the sixth and seventh cervical vertebrae. Volume 12, Number 3 / September, 1990

Cranial view of the course of the vertebral arteryover C1

Anterior arch of atlas

Vertebral arterySuperior articular fovea

Page 171: Comprehensive Neurosurgery Board Review Flash cards

©™©™

• Match the structures associated with this portion of the lateral ventricle:

1. Right foramen of Monro2. Left foramen of Monro3. Choroid plexus4. Thalamostriate vein5. Anterior cerebral artery6. Anterior septal vein7. Fornix8. Internal cerebral vein

NeuroanatomyQ?

A

BF

D

G

E

C

17

Page 172: Comprehensive Neurosurgery Board Review Flash cards

©™©™

NeuroanatomyA.• The correct answers are….• To obtain the answer to this question

and to view over 200 more comprehensive neuroanatomy questions please purchase the full product here !

A

B

F

D

G

E

C

Head of caudate

Thalamus

BOARD FAVORITE!

Comments:  Lateral ventricle anatomy, cerebral vascular anatomy